You are on page 1of 105

Finals Reviewer PARTNERSHIP 1st Sem; 2003

INTRODUCTION the Roman law, the law merchant and equity, and the common law courts. Brief Historical Background Governing law in our jurisdiction Development of partnership The earliest form of Before the new Civil Code, commercial or mercantile conducting business was the single entrepreneur partnerships were governed by the Code of Commerce ownership plan (one individual). Under this system, and non-commercial or civil partnerships by the old growth of the business was limited (limitation of capital, Spanish Civil Code. The new Civil Code superseded the skill or knowledge) and so partnership was developed. in toto the provisions in the Code of Commerce relating to Ancient origin of partnership as a business partnerships. Consequently, the new Civil Code organization Development of partnership often provisions are intended to provide all the rules regarding credited to the Romans. partnerships. There is no more distinction between commercial and civil partnerships. Romans. As early as 2300 B.C. the Babylonian system of The partnerships contemplated are those formed for laws provided for regulation of partnership. Commercial private interest or purpose. partnerships at that time were generally for single transactions or undertakings. Sources of our law on partnership Following the Babylonian period, there were clear-cut The Civil Code provisions on partnership were mostly references to partnerships in Jewish law. taken from the old Civil Code and from the USs Uniform Partnership Act and the . The relative newness of the law of partnership Some provisions were taken from the Code of Despite its long history of use, there is not a Commerce as well as from the opinions of civilians. New correspondingly long line of precedents and decisions rules were also formulated by the Code Commission. dealing with the subject. This is because English courts of justice scarcely dealt with the subject. Disputes between merchants were disposed of by special courts. The law of merchants In the Middle Ages, CHAPTER 1. GENERAL PROVISIONS merchants had a special and peculiar kind of law that was applicable to them and their legal affairs. Art. 1767. By the contract of partnership two or During this period, the common law courts of more persons bind themselves to contribute England were slow and methodically exact as to form. money, property, or industry to a common fund, Merchants moved more rapidly than the law and with the intention of dividing the profits among required speedier justice. Hence, the special courts. themselves. English law of partnership In time the special Two or more persons may also form a courts were discontinued and the law courts took over. partnership for the exercise of a profession. Chief Justice, Lord Mansfield sought to establish a common law for commercial matters. It was not until Concept of partnership the latter years of the 18th century that the law of partnership as we know it today began to assume both partnership (often called co -partnership) from the form and substance. viewpoint of a contract. In 1778, Lord Mansfield decided a case that dealt Partnership, however, has also been defined as: An association; with the rights of partnership. In 1794, William Watson 1.) A legal relation; wrote a text on partnership. 2.) A status; Beginning of law of partnership These two 3.) An organization; sources mark the beginning of printed precedents and 4.)

An entity; the publication of the principles of law in partnerships. 5.) A joint undertaking. Increase in use of partnership and complexity of [See6.) pp. 7-8 of De Leon (2002 Ed.) for full definitions. I think business brought forth a rapid succession of decisions on OK na if we just know the codal definition anyway.] partnerships. Partnership is a legal concept, but the determination American Uniform Acts Attempt made in US to drawn from the circumstances attending its creation and secure uniformity of state laws dealing with partnership. operation. The Uniform Partnership Act and the Uniform Limited Partnership Act helped to achieve this uniformity. The Civil law concept and American concept of partnership Uniform Partnership Act is similar with Englands distinguished Partnership Act of 1890. English settlers brought the Civil Code American partnership concept to their new country as part of the Basis of Partnership as a Partnership as a common law. concept contract: the relation: the Modern partnership law contains a combination of agreement itself result of the principles and concepts developed from three sources: out of which a contract or
Helen C. Arevalo 1 Section 3D

Finals Reviewer PARTNERSHIP 1st Sem; 2003


partnership is agreement; the 3.) Bilateral: two or more parties, reciprocal rights created. juridical relation. and obligations; Possession Entity theory: Aggregate 4.) Onerous: Each of the parties aspires to procure of separate Partnership has a theory: No a benefit through the giving of something; personality juridical separate juridical 5.) Commutative: undertaking of each partner is personality of its personality; considered the equivalent of that of the others; own, distinct and merely an 6.) Principal: does not depend upon some other separate from that extension of its contract for its existence/validity; of each of the members, a 7.) Preparatory: entered into as a means to an end. partners. conglomerate of A partnership contract, in its essence, is a contract of individuals. agency. Tax Partnerships, Partnership not Consequence except general taxed. Individual Essential features of partnership professional members 1.) There must be a valid contract; partnerships, are severally pay 2.) Parties must have legal capacity to enter into treated for income their income the contract; tax purposes as taxes, partnership 3.) Mutual contribution of money, property, or corporations and regarded merely industry to a common fund; subject to tax as as source of 4.) Object must be lawful; such. income. 5.) Primary purpose: to obtain profits and to divide them among the parties. General Professional Partnership It is also required that the articles of partnership Par. 2 relates to the exercise of a profession. must not be kept secret among the members;

[Profession: a group of men pursuing a learned art as a otherwise, the association shall have no legal personality common calling in the spirit of public service no less a public and shall be governed by the provisions relating to coservice because it may incidentally be a means of livelihood.] ownership.

The practice of a profession is not a business or an enterprise for profit. However, the law allows the joint Existence of a valid contract pursuit thereof by two or more persons as partners. It is Partnership relation fundamentally contractual the individual partners, and not the partnership, who Partnership is a voluntary relation created by engage in the practice of the profession and are agreement of the parties. responsible for their own acts as such. The law does not Actually, the partnership relation is not the contract allow the practice of a profession as a corporate entity. itself, but the result of the contract. Personal qualifications for such practice cannot be Form The relation is evidenced by the terms of possessed by a corporation. the contract which may be oral or written, express or implied from the acts and declarations of the parties. Partnership for the practice of law Articles of partnership While the partnership A mere association for non-business purpose - relation may be informally created and its existence Right to practice law not a constitutional right but a proved by the manifestations of the parties, it is privilege or franchise. It cannot be likened to customary to embody the terms of the association in a partnerships formed by other professionals or for written document known as Articles of Partnership. business. Requisites Since partnership is contractual, all It is not formed for the purpose of carrying on trade the essentials of a valid contract must be present: or business or of holding property. Thus, use of a nom 1.) Consent and capacity of parties; de plume, assumed, or trade name is improper. 2.) Object; Distinguished from business It is intimately and 3.) Cause. peculiarly related to the administration of justice; not a A person cannot enter into a contract of partnership mere money-making trade. Primary characteristics solely by himself; there must be two contracting parties. which distinguish it: For a partnership to be valid, there must be a valid 1.) Duty of public service; consideration existing as between the partners. Each 2.) Relation as an officer of court to the surrenders to the partnership some sort of contribution. administration of justice; Partnership relation fiduciary in nature

3.) Highly fiduciary relation to clients; Partnership is a form of voluntary association entered 4.) Relation to colleagues at the bar characterized into by the associates. It is a personal relation in which by candor, fairness, etc. the element of delectus personae exists, involving as it does trust and confidence between the partners. Characteristic elements of partnership Membership requires the consent of all. Its fiduciary 1.) Consensual: perfected by mere consent nature and the liability of each partner for the acts of (express/implied); the others require that each person be granted the right 2.) Nominate: has a special name/designation in our to choose with whom he will be associated with. law;
Helen C. Arevalo 2 Section 3D

Finals Reviewer PARTNERSHIP 1st Sem; 2003


Among partners, mutual agency arises and the Proof of contribution Proof is necessary that doctrine of delectus personae allows them to have the there be contribution of money, property, or industry to power to dissolve the partnership. Any partner may, at a common fund with the intention of dividing the income his sole pleasure, dictate dissolution at will. He must, or profits obtained therefrom. If only one partner gives, however, act in good faith or hell be liable for damages. no enforceable contract exists. Application of principles of estoppel Partnership liability may be imposed when one holds Legality of the object himself out, or permits himself to be held out, as a The object is unlawful when it is contrary to law, partner. There is no actual or legal partnership relation morals, good customs, public order, or public policy. If but merely a partnership liability imposed by law in purpose unlawful, no partnership can arise as the favor of third persons. contract is inexistent and void ab initio. Legal capacity of the parties to enter into the Purpose to obtain profits contract The very reason for existence of partnership Individuals General rule: any person who is The idea of obtaining pecuniary profit or gain is the very capable of entering into contractual relations may be a reason for the existence of a partnership. partner. The following cannot: Need only be the principal, not exclusive aim 1.) Minors; pecuniary profit need not be the exclusive aim. It is 2.) Insane or demented persons; sufficient that it is the principal purpose even if there 3.) Deaf-mutes who do not know how to write; are, incidentally, other ends. 4.) Persons who are suffering from civil interdiction; 5.) Incompetents who are under guardianship. Sharing of profits Persons who are prohibited from giving each other Not necessarily in equal shares There must be any donation cannot enter into a universal partnership. intention to divide the profits but not necessarily in Partnerships There is no prohibition against a equal shares. There must be a joint interest in the partnership being a partner in another partnership. All profits. the members of the constituent partnerships will be held A stipulation which excludes a partner from any liable to the creditors of the combined partnership. participation in the profits is void. Corporations Unless authorized by statute or by Not conclusive evidence of partnership The its charter, a corporation is without capacity or power to sharing of profits is merely presumptive and not enter into a contract of partnership. This is based on conclusive evidence of partnership. public policy, since in a partnership the corporation would be bound by the acts of persons who are not its Sharing of losses duly appointed and authorized agents and officers, Necessary corollary of sharing in profits The which would be entirely inconsistent with the policy that right to share in the profits carries with it the duty to the corporation shall manage its own affairs separately contribute to the losses, if any. A community in losses is and exclusively. a necessary corollary of a participation in profits. Exceptions: Agreement not necessary It is not necessary for 1.) Joint ventures where the nature of the venture is the parties to agree on a system of sharing losses, for in line with the business authorized by its the obligation is implied from the partnership relation. If charter. only the share of each partner in the profits has been 2.) Partnership agreement provides that the two agreed upon, the share of each in the losses shall be in partners will manage the partnership so that the the same proportion. management of corporate interest is not Generally, a stipulation which excludes one or more surrendered. partners from any share in the profits or losses is void. 3.) Entry of foreign corp as a limited partner in a

limited partnership merely for investment Art. 1768. The partnership has a juridical purposes. personality separate and distinct from that of each of the partners even in case of failure to comply Contribution of money, property, or industry to a with the requirements of art. 1772, 1st paragraph. common fund Existence of proprietary interest The partners Partnership, a juridical person must have a proprietary interest in the business or As an independent juridical person, a partnership undertaking, that is, they must contribute capital which may enter into contracts, acquire and possess property may be money, or property, or their services, or both, to of all kinds in its name, as well as incur obligations and the common business. bring civil or criminal actions.
Money Legal tender in the Phils. Thus, a partnership may be declared insolvent even if Property Real or personal, corporeal or incorporeal. Industry Active cooperation, the work of the party the partners are not. It may enter into contracts and associated. may sue and be sued in its firm name or by its duly

authorized representative. It is sufficient that service of summons be served on any partner.

Helen C. Arevalo 3 Section 3D

Finals Reviewer PARTNERSHIP 1st Sem; 2003


Partners cannot be held liable for the obligations of 1769 shall apply. This article seeks to exclude from the the partnership unless it is shown that the legal fiction of category of partnership certain features enumerated a different juridical personality is being used for a therein which, by themselves, are not indicative of the fraudulent, unfair or illegal purpose. existence of a partnership. Effect of failure to comply with statutory Persons not partners as to each other requirements Persons who are partners as between themselves are Under art. 1772 Partnership still acquires partners as to third persons. Generally, the converse is personality despite failure to comply with the true: if they are not partners between themselves, they requirements of execution of public instrument and cannot be partners as to third persons. registration of name in SEC. Partnership is a matter of intention, each partner Under arts. 1773 and 1775 Partnership with giving his consent to become a partner. However, immovable property contributed, if without requisite whether a partnership exists between the parties is a inventory, signed and attached to public instrument, factual matter. Where parties declare they are not shall not acquire any juridical personality because the partners, this, as a rule, settles the question between contract itself is void. This is also true for secret themselves. But where a person misleads third persons associations or societies. into believing that they are partners in a non-existent partnership, they become subject to liabilities of To organize a partnership not an absolute right partners (doctrine of estoppel). It is but a privilege which may be enjoyed only under Whether or not the parties call their relationship or such terms as the State may deem necessary to impose. believe it to be a partnership is immaterial. Thus, with the exception of partnership by estoppel, a partnership Art. 1769. In determining whether a cannot exist as to third persons if no contract of partnership exists, these rules shall apply: partnership has been entered into between the parties 1.) Except as provided by art. 1825, persons themselves. who are not partners as to each other are not partners as to 3rd persons; Co-ownership or co-possession 2.) Co-ownership or co-possession does not of There is co-ownership whenever the ownership of an itself establish a partnership, whether such undivided thing or right belongs to different persons. co-ownership or co-possessors do or do not Clear intent to derive profits from operation of share any profits made by the use of the business Co-ownership does not of itself establish the property; existence of a partnership, although it is one of its 3.) The sharing of gross returns does not of essential elements. This is true even if profits are itself establish a partnership, whether or derived from the joint ownership. The profits must be not the persons sharing them have a joint derived from the operation of business by the members or common right or interest in any property of the association and not merely from property from which the returns are derived; ownership. 4.) The receipt by a person of a share of the The law does not imply a partnership between coprofits of a business is prima facie evidence owners because of the fact that they develop or operate that he is a partner in the business, but no a common property, since they may rightfully do this by such inference shall be drawn if such virtue of their respective titles. There must be a clear profits were received in payment: intent to form a partnership. a.) As a debt by installments or Existence of fiduciary relationship Partners otherwise; have a well-defined fiduciary relationship between them. b.) As wages of an employee or rent to Co-owners do not. Should there be dispute, the remedy a landlord; of partners is an action for dissolution, termination and c.) As an annuity to a widow or accounting. For co-owners it would be one, for instance, representative of a deceased for non-performance of contract. partner; People can become co-owners without a contract but

d.) As interest on a loan, though the they cannot become partners without one. amount of payment vary with the Persons living together without benefit of profits of the business; marriage Property acquired governed by rules on coe.) As the consideration for the sale of a ownership. goodwill of a business or other property by installments or Sharing of gross returns otherwise. Not even presumptive evidence of partnership The mere sharing of gross returns alone does not even Rules to determine existence of partnership constitute prima facie evidence of partnership, since in a In general, to establish the existence of a partnership, the partners share profits after satisfying all partnership, all of its essential features or characteristics of the partnerships liabilities. must be shown as being present. In case of doubt, art.
Helen C. Arevalo 4 Section 3D

Finals Reviewer PARTNERSHIP 1st Sem; 2003


Reason for the rule Partner interested in both Tests and incidents of partnership failures and successes; it is the chance of loss or gain In determining whether a partnership exists, it is that characterizes a business. Where the contract important to distinguish between tests or indicia and requires a given portion of gross returns to be paid over, incidents of partnership. the portion is paid over as commission, wages, rent, etc. Only those terms of a contract upon which the Where there is evidence of mutual management parties have reached an actual understanding, either Where there is further evidence of mutual expressly or impliedly, may afford a test by which to management and control, partnership may result. ascertain the legal nature of the contract. Some of the typical incidents of a partnership are: Receipt of share in the profits 1.) The partners share in profits and losses. Strong presumptive evidence of partnership 2.) They have equal rights in the mgt and conduct An agreement to share both profits and losses tends of the partnership business. strongly to establish the existence of a partnership. It is 3.) Every partner is an agent of the partnership, and not conclusive, however, just prima facie and may be entitled to bind the others by his acts. He may rebutted by other circumstances. also be liable for the entire partnership When no such inference will be drawn Under obligations. par. 4 of art. 1769, sharing of profits is not prima facie 4.) All partners are personally liable for the debts of evidence of partnership in the cases enumerated under the partnership with their separate property subsections (a) (e). In these cases, the profits are not except that limited partners are not bound shared as partner but in some other respects or beyond the amount of their investment. purpose. 5.) A fiduciary relation exists between the partners. The basic test of partnership is whether the business 6.) On dissolution, the partnership is not is carried on in behalf of the person sought to be held terminated, but continues until the winding up of liable. partnership is completed. Sharing of profits as owner It is not merely the Such incidents may be modified by stipulation of the sharing of profits, but the sharing of them as co-owner partners. of the business or undertaking, that makes one partner. Test: Does the recipient have an equal voice as Partnership distinguished from a labor union proprietor in the conduct and control of the business? Partnership Labor Union Does he own a share of the profits as proprietor of the Purpose To enable its members, Collective business producing them? as principals, to conduct bargaining; One must have an interest with another in the profits a lawful business, trade, dealing with of a business as profits. or profession for employers pecuniary gain of concerning terms Burden of proof and presumption partners, and no one and conditions of The burden of proving the existence of a partnership may become a partner employment. rests on the party having the affirmative of that issue. w/o consent of all. The existence of a partnership must be proved and will not be presumed. Partnership distinguished from a business trust The law presumes that those acting as partners have entered into a contract of partnership. Where the law Partnership Trust presumes the existence of partnership, the burden of Agency All members are Trustee is only a proof is on the party denying its existence. principals and principal and is not When a partnership is shown to exist, the agents for each an agent. presumption is that it continues and the burden of proof other. is on the person asserting its termination. Ownership Partners are co-Beneficiary has One who alleges partnership cannot prove it merely of property owners of specific equitable ownership by evidence of an agreement using the term partner. partnership of property while Non-use of the term, however, is entitled to weight. property. trustee owns legal The question of whether a partnership exists is not title to such always dependent upon the personal arrangement or property. understanding of the parties. Parties intending to do a

thing which in law constitutes partnership are partners. Legal intention is the crux of partnership. Parties may Partnership distinguished from co-ownership call themselves partners but their contract may be Partnership Co-ownership adjudged something quite different. Conversely, parties Creation Always created by Generally created may expressly state that theirs in not a partnership yet contract, whether by law. It may the law may determine otherwise on the basis of legal express or implied. exist even w/o a intent. However, courts will be influenced to some contract. extent by what the parties call their contract. Juridical Has a juridical No separate personality personality separate juridical
Helen C. Arevalo 5 Section 3D

Finals Reviewer PARTNERSHIP 1st Sem; 2003


and distinct from personality. one or more belongs to both that of each partner. appointed jointly, husbands Purpose Realization of profits. Common managers. decision prevails enjoyment of a in case of thing or right. disagreement. Duration No limitation under 10 year limit. Disposition Whole interest of a Share of each the law. of shares partner may be spouse cannot be Disposal of Partner may not Co-owner may disposed of disposed of interests dispose of his assign w/o without consent of during marriage individual interest in consent of other others. even w/ consent the partnership so as co-owners. of other. to make the assignee partner w/o consent of all. Partnership distinguished from a voluntary Power to Partner may bind Co-owner cannot association act w/ 3rd partnership. represent co- Partnership Voluntary persons ownership. association Effect of Death of partner Death of co-Juridical Meron. Wala. death results in dissolution. owner does not personality necessarily Purpose Always for Not. dissolve co-pecuniary profit. ownership. Contributions Contribution of Although fees of members capital, either are usually money, property collected from Partnership distinguished from conjugal or services. the members, no partnership of gains contribution of Ordinary/ Conjugal capital. business partnership of Liability of Partnership is Members are partnership gains members the one liable in individually liable Parties Created by Arises in case the the 1st place for for debts of voluntary future spouses debts of the firm. association. agreement of 2 or a man and a more partners woman agree belonging to either that it shall Partnership distinguished from a corporation sex. govern their Partnership Corporation property relations Manner of Mere agreement of Law or operation during marriage. creation parties. of law. Laws which Stipulation of the Law. Number of At least two. At least five. govern parties. incorporators Juridical Meron. Wala. Commence-From moment of From date of personality ment of execution of issuance of Commence-Begins from Commences juridical contract of certificate of ment moment of precisely on the personality partnership. incorporation by execution of date of the SEC. contract, unless celebration of the Powers Partnership may Corp can only otherwise marriage and any exercise any power exercise powers stipulated. stipulation to authorized by expressly contrary is void. partners provided granted by law Purpose To obtain profits. To regulate the not contrary to or implied from property relations morals, good those granted or of husband and customs, etc. incident to its wife during existence. marriage. Management When mgt not Power to manage Distribution Divided according Equal. agreed upon, every vested in board of profits to agreement or in partner is agent. of directors or proportion to trustees.

capital Effect of Partner can sue co-Suit against contributions. mismanage-partner who member of board Management Shared equally by Although ment mismanages. of directors or partners unless administration trustees who
Helen C. Arevalo 6 Section 3D

Finals Reviewer PARTNERSHIP 1st Sem; 2003


mismanages confiscation of the instruments and effects of a must be in name crime. of corp. Right of Wala. Meron. Object or purpose of partnership succession The provision of the 1st paragraph reiterates 2 Extent of Partners (except Stockholders essential elements of a contract of partnership: 1. liability to 3rd limited partners) liable only to legality of the object; and 2. community of benefit or persons are liable extent of the interest of the partners. personally and shares The parties possess absolute freedom to choose the subsidiarily subscribed by transaction or transactions they must engage in. The (sometimes them. only limitation is that the object must be lawful and for solidarily) for the common benefit of the members. partnership debts The illegality of the object will not be presumed; it to 3rd persons. must appear to be of the essence of the relationship. Transfer-Partner cannot Stockholder has ability of transfer his interest generally the Effects of an unlawful partnership interest so as to make right to transfer 1.) The contract is void ab initio and the partnership transferee a his shares w/o never existed in the eyes of the law; partner w/o prior consent of 2.) The profits shall be confiscated in favor of the consent of others. the other government; Delectus stockholders. 3.) The instruments or tools and proceeds of the personarum. crime shall also be forfeited in favor of the Term of Any period of time 50 years max. government; existence stipulated by the extendible to 4.) The contributions of the partners shall not be partners. another 50. confiscated unless they fall under #3. Firm name Limited partnership Corp may adopt A partnership is dissolved by operation of law upon reqd to add word any firm name the happening of an event which makes it unlawful. Ltd. provided not A judicial decree is not necessary to dissolve an same or similar unlawful partnership. However, advisable that judicial to any registered decree be secured. 3rd persons who deal w/ partnership firm name. w/o knowledge of illegal purpose are protected. Dissolution May be dissolved at Can only be any time by the will dissolved with Right to return of contribution where partnership of any or all of the consent of State. is unlawful partners. Partners must be reimbursed the amount of their Governing Civil Code. Corporation respective contributions. The partner who limits himself law Code. to demanding only the amount contributed by him need not resort to the partnership contract on which to base his claim or action. Since the purpose for which the Similarities between a partnership and a contribution was made has not come into existence, the corporation mgr or administrator must return it, and he who has 1.) Both have juridical personality separate and paid his share is entitled to recover it. distinct from that of the individuals composing it; Right to receive profits where partnership is 2.) Both can only act through its agents; unlawful 3.) Both are organizations composed of an Law does not permit action for obtaining earnings aggregate of individuals; from an unlawful partnership because for that purpose, 4.) Both distribute profits to those who contribute the partner will have to base his action upon the capital to the business; partnership contract, which is null and without legal 5.) Both can only be organized where there is a law existence by reason of its unlawful object; and it is selfauthorizing is organization; evident that what does not exist cannot be a cause of 6.) Partnerships are taxable as corporations. action. Profits earned do not constitute or represent the Art. 1770. A partnership must have a lawful partners contribution. He must base his claim on the object or purpose, and must be established for the contract which is void.

common benefit or interest of the partners. It would be immoral and unjust for the law to permit When an unlawful partnership is dissolved by a a profit from an industry prohibited by it. judicial decree, the profits shall be confiscated in The courts will refuse to recognize its existence, and favor of the State, without prejudice to the will not lend their aid to assist either of the parties provisions of the Penal Code governing the thereto in an action against each other. Therefore, there can be no accounting demanded of a partner for the
Helen C. Arevalo 7 Section 3D

Finals Reviewer PARTNERSHIP 1st Sem; 2003


profits which may be in his hands, nor can recovery be When partnership agreement covered by the had. Statute of Frauds An agreement to enter in a partnership at a future time, which by its terms is not to Effect of partial illegality of partnership business be performed w/in a year from the making thereof is Where a part of the business is legal and part illegal, covered by the Statute of Frauds. Such agreement is an account of that which is legal may be had. unenforceable unless it is in writing or at least evidenced Where, w/o the knowledge or participation of the by some note or memorandum. partners, the firms profits in a lawful business have been increased by wrongful acts, the innocent partners Partnership implied from conduct are not precluded as against the guilty partners from Binding effect Existence of partnership may be recovering their share of the profits. implied from the acts or conduct of the parties, as well as from other declarations, and such implied contract Effect of subsequent illegality of partnership would be as binding as a written and express contract. business Ascertainment of intention of parties In Contract will not be nullified. Where the business for determining whether a particular transaction constitutes which the partnership is formed is legal when the a partnership, as between the parties, the intention as partnership is entered into, but afterward becomes disclosed by the entire transaction, and as gathered illegal, an accounting may be had as to the business from the facts and from the language employed by the transacted prior to such time. parties as well as their conduct, should be ascertained. Conflict between intention and terms of Community of interest between the partners for contract If the parties intend a general partnership, business purposes they are general partners although their purpose is to The salient features of an ordinary partnership are a avoid the creation of such a relation. community of interest in profits and losses, a community of interest in the capital employed, and a community of Art. 1772. Every contract of partnership having power in administration. a capital of three thousand pesos or more, in This community of interest is the basis of the money or property, shall appear in a public partnership relation. However, although every instrument, which must be recorded in the Office partnership is founded on a community of interest, of the Securities and Exchange Commission. every community of interest does not necessarily Failure to comply with the requirements of the constitute a partnership. preceding paragraph shall not affect the liability of Property used in the business may belong to one or the partnership and the members thereof to third more partners, so that there is no joint property, other persons. than joint earnings. To state that partners are co-owners of a business is to state that the have the power if Registration of partnership ultimate control. But partners may agree upon Partnership with capital of P3,000 or more 2 concentration of management, leaving some of their requirements: members entirely inactive or dormant. 1.) The contract must appear in a public instrument; Only one of these features, profit-sharing, seems to 2.) It must be recorded or registered w/ the SEC. be absolutely essential. But a mere sharing of profits of However, failure to comply w/ the above itself does not of necessity constitute a partnership. requirements does not prevent the formation of the The court must consider all the essential elements in partnership or affect its liability and that of the partners light of the facts of the particular case before deciding to 3rd persons. But any partner is granted the right by whether a partnership exists. law to compel each other to execute the contract in a public instrument. Art. 1771. A partnership may be constituted in Purpose of registration Registration is necessary any form, except where immovable property or as a condition for the issuance of licenses to engage in real rights are contributed thereto, in which case a business and trade. In this way, the tax liabilities of big

public instrument shall be necessary. partnerships cannot be evaded and the public can determine more accurately their membership and capital Form of partnership contract before dealing with them. General rule No special form required for validity When partnership considered registered The or existence of the contract of partnership. Contract may objective of the law is to make the recorded instrument be made orally or in writing regardless of the value of open to all and to give notice thereof to interested the contributions. parties. Where immovable property or real rights are This objective is achieved from the date the contributed Execution of public instrument necessary partnership papers are presented to and left for record for validity of contract of partnership. To affect 3rd in the Commission. This is the effective date of persons, the transfer of real property to the partnership registration. If the certificate of recording is issued on a must be duly registered in the Registry of Property. subsequent date, its effectivity retroacts to date of presentation.
Helen C. Arevalo 8 Section 3D

Finals Reviewer PARTNERSHIP 1st Sem; 2003


Secret partnerships without juridical personality Art. 1773. A contract of partnership is void, Partnership relation is created only by the voluntary whenever immovable property is contributed agreement of the partners. It is essential that the thereto, if an inventory of said property is not partners are fully informed not only of the agreement made, signed by the parties, and attached to the but of all matters affecting the partnership. Secret public instrument. partnerships are not by nature partnerships. Secret partnerships shall be governed by the Partnership with contribution of immovable provisions relating to co-ownership. property Where immovable property contributed, failure to Importance of giving publicity to articles of comply w/ the following requisites will render the partnership partnership contract void: It is essential that the arts of partnership be given 1.) The contract must be in a public instrument; publicity for the protection not only of the members 2.) An inventory of the property contributed must themselves but also 3rd persons from fraud and deceit. A be made, signed by the parties, and attached to member who transacts business for the secret the public instrument. partnership in his own name becomes personally bound Art. 1773 is intended primarily to protect 3rd persons. to 3rd persons unaware of the existence of such W/ regard to 3rd persons, a de facto partnership or association. partnership by estoppel may exist. There is nothing to Partnership liability may still result, however, in cases prevent the court from considering the partnership of estoppel. agreement an ordinary contract from which the parties rights and obligations to each other may be inferred and Art. 1776. As to its object, a partnership is enforced. either universal or particular. As regards the liability of the partners, a When inventory is not required partnership may be general or limited. An inventory is required only whenever immovable property is contributed. If not contributed or if personal Classifications of partnership property, no inventory required. As to extent of its subject matter 1.) Universal partnership. (Art. 1777) Importance of making inventory of real property in a.) Universal partnership of all present a partnership property. (Art. 1778) An inventory is very important in a partnership to b.) Universal partnership of profits. (Art. show how much is due from each partner to complete 1780) his share in the common fund and how much is due to 2.) Particular partnership. (Art. 1783) each of them in case of liquidation. As to liability of the partners The execution of a public instrument of partnership 1.) General partnership: one consisting of general would be useless if there is no inventory of immovable partners who are liable pro rata and subsidiarily property contributed because w/o its description and and sometimes solidarily w/ their separate designation, the instrument cannot be subject to property for partnership debts. inscription in the Registry of Property, and the 2.) Limited partnership: one formed by two or more contribution cannot prejudice 3rd persons. persons having as members one or more general partners and one or more limited partners, the Art. 1774. Any immovable property or an latter not being personally liable for the interest therein may be acquired in the obligations of the partnership. partnership name. Title so acquired can be As to duration conveyed only in the partnership name. 1.) Partnership at will: one in w/c no time is specified and is not formed for a particular Acquisition or conveyance of property by undertaking or venture and w/c may be partnership terminated at any time by mutual agreement of Since partnership has juridical personality of its own, the partners, or by the will of any one partner it may acquire immovable property in its own name. alone; or one for a fixed term or particular Title so acquired can be conveyed only in the undertaking w/c is continued after the end of the partnership name. term or undertaking w/o express agreement. 2.) Partnership with a fixed term: one w/c the term

Art. 1775. Associations and societies, whose for w/c the partnership is to exist is fixed or articles are kept secret among the members, and agreed upon or one formed for a particular wherein any one of the members may contract in undertaking. his own name with third persons, shall have no As to the legality of its existence juridical personality, and shall be governed by the 1.) De jure partnership: one w/c has complied w/ all provisions relating to co-ownership. the legal requirements for its establishment.

Helen C. Arevalo 9 Section 3D

Finals Reviewer PARTNERSHIP 1st Sem; 2003


2.) De facto partnership: one w/c has failed to 10.) Subpartner: one who, not being a member of comply w/ all the legal requirements for its the partnership, contracts w/ a partner w/ establishment. reference to the latters share in the partnership. As to representation to others Other classifications 1.) Ordinary or real partnership: one w/c actually 1.) Ostensible partner: one who takes active part exists among the partners and also as to 3rd and known to the public as a partner. persons. 2.) Secret partner: one who takes active part in the 2.) Ostensible partnership or partnership or business but is not known to be a partner by partnership by estoppel: one w/c in reality is not outside parties nor held out as a partner by the a partnership, but is considered a partnership other partners. He is an actual partner. only in relation to those who, by their conduct or 3.) Silent partner: one who does not take any active admission, are precluded to deny or disprove its part in the business although he may be known existence. to be a partner. As to publicity 4.) Dormant partner: (a.k.a. sleeping partner) one 1.) Secret partnership: one wherein the existence of who does not take active part in the business certain persons as partners is not avowed or and is not known or held out as a partner. He made known to the public by any of the would be both a silent and a secret partner. partners. 5.) Original partner: one who is a member of the 2.) Open or notorious partnership: one whose partnership from the time of its organization. existence is avowed or made known to the 6.) Incoming partner: a person lately, or about to public by the members of the firm. be, taken into an existing partnership as a As to purpose member. 1.) Commercial or trading partnership: one formed 7.) Retiring partner: one withdrawn from the for the transaction of business. partnership; a withdrawing partner. 2.) Professional or non-trading partnership: one formed for the exercise of a profession. Art. 1777. A universal partnership may refer to all the present property or to all the profits. Kinds of partners Under the Civil Code Art. 1778. A partnership of all present property 1.) Capitalist partner: one who contributes money is that in which the partners contribute all the or property to the common fund. property which actually belongs to them to a 2.) Industrial partner: one who contributes only his common fund, with the intention of dividing the industry or personal service. same among themselves, as well as all the profits 3.) General partner: (a.k.a. real partner) one they may acquire therewith. whose liability to 3rd persons extends to his separate property. Art. 1779. In a universal partnership of all 4.) Limited partner: (a.k.a. special partner) one present property, the property which belongs to whose liability to 3rd persons is limited to his each of the partners at the time of the constitution capital contribution. of the partnership, becomes the common property 5.) Managing partner: (a.k.a. general or real of all the partners, as well as all the profits which partner how confusing!) one who manages the they may acquire therewith. affairs or business of the partnership. A stipulation for the common enjoyment of any 6.) Liquidating partner: one who takes charge of the other profits may also be made; but the property winding up of partnership affairs upon which the partners may acquire subsequently by dissolution. inheritance, legacy or donation cannot be included 7.) Partner by estoppel: (a.k.a. partner by in such stipulation, except the fruits thereof. implication or nominal partner or even quasipartner) one who is not really a partner but is Universal partnership of all present property liable as a partner for the protection of innocent explained 3rd persons. He is one represented as being a A universal partnership of profits is one w/c partner but who is not so between the partners comprises all that the partners may acquire by their themselves. industry or work during the existence of the partnership 8.) Continuing partner: one who continues the and the usufruct of movable or immovable property w/c business of a partnership after it has been each of the partners may possess at the time of the dissolved by reason of the admission of a new celebration of the contract. partner, or the retirement, death or expulsion of In this kind of partnership, the following become the one or more partners. common property of all the partners:

9.) Surviving partner: one who remains after a 1.) Property w/c belonged to each of them at the partnership has been dissolved by the death of time of the constitution of the partnership; any partner. 2.) Profits w/c they may acquire from the property contributed.
Helen C. Arevalo 10 Section 3D

Finals Reviewer PARTNERSHIP 1st Sem; 2003


Contribution of future property Limitations upon the right to form a partnership General rule: future properties cannot be Persons who are prohibited by law to give donations contributed. The very essence of the contract of cannot enter into a universal partnership for the reason partnership that the properties contributed be included that each of the partners virtually makes a donation. To in the partnership requires the contribution of things allow it would be permitting them to do indirectly what determinate. The position of a partner is like that of a the law expressly prohibits. donor, and donations cannot comprehend future A partnership formed in violation of this article is null property. Thus, property subsequently acquired by 1. and void. Consequently, no legal personality is acquired. inheritance; 2. legacy; or 3. donation cannot be included A husband and wife, however, may enter into a by stipulation except the fruits thereof. Hence, any particular partnership or be members thereof. stipulation including property so acquired is void. Relevant provisions: Profits from other sources (not from properties 1.) Art. 87: Donations between spouses during contributed) will become common property only is marriage void, except moderate gifts on theres a stipulation. occasion of family rejoicing. Also applies to those living together as husband and wife w/o valid Art. 1780. A universal partnership of profits marriage. comprises all that the partners may acquire by 2.) Art. 739: The following donations are void: their industry or work during the existence of the a.) Those made between persons who are partnership. guilty of adultery or concubinage at the Movable or immovable property which each of time of the donation (no need for the partners may possess at the time of the conviction; preponderance of evidence celebration of the contract shall continue to only required); pertain exclusively to each, only the usufruct b.) Those made between persons found passing to the partnership. guilty of the same criminal offense, in consideration thereof; Universal partnership of profits explained c.) Those made to a public officer or his A universal partnership of profits is one w/c wife, descendants and ascendants, by comprises all that the partners may acquire by their reason of his office. industry or work during the existence of the partnership and the usufruct of movable or immovable property w/c Art. 1783. A particular partnership has for its each of the partners may possess at the time of the object determinate things, their use or fruits, or a celebration of the contract. specific undertaking, or the exercise of a Ownership of present and future property The profession or vocation. partners retain their ownership over their present and future property. What passes to the partnership are the Particular partnership explained profits or income and the use or usufruct of the same. A particular partnership is one w/c is neither a Consequently, upon dissolution, such property is universal partnership of present property nor a universal returned to the partners who own it. partnership of profits. Profits acquired through chance Since the law The fundamental difference between a universal only speaks of profits w/c the partners may acquire by partnership and a particular partnership lies in the scope their industry or work, profits acquired purely by chance of their subject matter or object. In the former, the are not included. object is vague and indefinite, contemplating a general Fruits of property subsequently acquired Fruits business w/ some degree of continuity, while in the of property subsequently acquired by the partners do latter, it is limited and well-defined, being confined to an not belong to the partnership. Such profits, however, undertaking of a single, temporary, or ad hoc nature.

may be included by express stipulation. Business of partnership need not be continuing in Art. 1781. Articles of universal partnership, nature entered into without specification of its nature, The carrying on of a business of a continuing nature only constitute a universal partnership of profits. is not essential to constitute a partnership. An agreement to undertake a particular piece of work or a Presumption in favor of universal partnership pf single transaction or a limited number of transactions profits and immediately divide the resulting profits would seem Reason for presumption: universal partnership of to fall w/in the meaning of the term partnership as profits imposes less obligations on the partners, since used in the law. they preserve the ownership of their separate property. Rule under American law The above is not true under the Uniform Partnership Act w/c does not include Art. 1782. Persons who are prohibited from joint ventures w/c exists for a single transaction or a giving each other any donation or advantage limited number of transactions. cannot enter into a universal partnership.
Helen C. Arevalo 11 Section 3D

Finals Reviewer PARTNERSHIP 1st Sem; 2003


Joint venture While a joint venture is not a formal Art. 1784. A partnership begins from the partnership in the legal or technical sense, both are moment of the execution of the contract, unless it governed, subject to certain qualifications, practically by is otherwise stipulated. the same rules or principles of partnership. This is logical since in a joint venture, like in a partnership, Commencement and term of partnership there is a community of interest in the business and a As a consensual contract, a partnership exists from mutual right of control and an agreement to share the moment of the celebration of the contract. Its jointly in profits and losses. registration with the SEC is not essential to give it Corporation as a partner While under the juridical personality. Philippine Civil Code, a joint venture is a form of The birth and life of a partnership is predicated on partnership w/ a legal personality separate and distinct the mutual desire and consent of the parties. Unlike from the parties composing it, and should thus be corporations, no time limit is prescribed by law for a governed by the law of partnership, the Supreme Court partnerships lifetime. Partners may fix in their contract has recognized the distinction between these two any term. business forms, and has held that although a corporation cannot enter into a partnership contract, it Rules governing partnership relation may, however, engage in a joint venture if the nature of What is necessary for the existence of partnership is the venture is authorized by its charter. that the essential requisites of a contract of partnership are present even when the partners have not yet actually started business/given contributions, etc. CHAPTER 2. OBLIGATIONS OF THE PARTNERS Where a partnership relation results, the law itself fixes the incidents and consequences of this relation if SECTION 1. OBLIGATIONS OF THE PARTNERS the parties fail to do so. This is true even if parties call AMONG THEMSELVES their relation something different or state that they are not partners. Relations created by a contract of partnership 1.) Relations among the partners themselves; Executory agreement of partnership 2.) Relations of the partners with the partnership; The above rule on commencement of a partnership is 3.) Relations of the partnership with third persons; not absolute. 4.) Relations of the partners with third persons. Future partnership The partners may stipulate some other date for the commencement of the Rights and obligations, in general, or partners partnership. There can be a future partnership which at inter se the moment has no juridical existence yet. Partnership relationship essentially one of If it is not to start within a year of the making of the mutual trust and confidence Each partner is a contract, it should be in writing in order to be trustee and a cestui que trust at the same time. He is a enforceable (Statute of Frauds). trustee to the extent that his duties bind him, a cestui Agreement to create partnership A partnership que trust as far as the duties that rest on his co-in fact cannot be predicated on an agreement to enter partners. into a co-partnership at a future day unless it is shown The many particular rights and duties are but aspects that such an agreement was actually consummated. So of the broad fiduciary relation. long as the agreement remains executory the Fiduciary relationship remains until partnership partnership is inchoate. terminated The relation of trust applies also to The death of either party to an executory agreement matters concerned with the formation of the partnership prevents the formation of the firm, since such and when a partnership is dissolved, the assets of the agreement is based on the continuance of the life of partnership must still be managed in accordance with each. this fiduciary principle. The fiduciary obligation of a Failure to agree on material terms May prevent partner remains until the relationship is terminated and any rights and obligations from arising on either side for the equities between the partners adjusted and lack of complete contract.

satisfied. Relationship in a limited partnership The rights Art. 1785. When a partnership for a fixed term and obligations of the partners as to each other are or particular undertaking is continued after the provided on the theory that a partner is both a principal termination of such term or particular undertaking and an agent in relation to his co-partners. But the without any express agreement, the rights and relationship between a limited partner and the other duties of the partners remain the same as they partners in a limited partnership does not involve the were at such termination, so far as is consistent element of trust and confidence, as in the case of a with a partnership at will. general partnership. A continuation of the business by the partners or such of them as habitually acted therein during the term, without any settlement or liquidation of
Helen C. Arevalo 12 Section 3D

Finals Reviewer PARTNERSHIP 1st Sem; 2003


the partnership affairs, is prima facie evidence of a 1.) To contribute at the beginning of the partnership continuation of the partnership. or at the stipulated time the money, property, or industry he had promised; Continuation of partnership beyond fixed term 2.) To answer for eviction in case the partnership is A partnership with a fixed term is one which the term deprived of the determinate property of its existence has been agreed upon expressly (definite contributed; period) or impliedly (particular enterprise or 3.) To answer to the partnership for the fruits of the transaction). The expiration of such term or property the contribution of which he delayed, accomplishment of undertaking will cause automatic from the date they should have been contributed dissolution. up to the time of actual delivery; Rights and duties of partners Partnership may 4.) To preserve said property with the diligence of a be extended or renewed by the partners by express good father of a family pending delivery to the agreement, written or oral, or impliedly, by the mere partnership; continuation of the business after the termination pf 5.) To indemnify the partnership for any damage such term or particular undertaking without any caused to it by the retention of the same or by settlement or liquidation. In such case, the rights and the delay in its contribution. duties remain the same. With such continuation, the partnership is dissolved Effect of failure to contribute property promised and a new one, a partnership at will, is created by Failure to contribute makes the partner ipso jure a implied agreement the continued existence of which will debtor of the partnership even in the absence of any depend upon the mutual desire and consent of the demand. partners. Remedy: not rescission but an action for specific Dissolution of partnership Any one of the performance (to collect what is owing) with damages partners may, at his sole pleasure, dictate a dissolution and interest. of a partnership at will. He must, however, act in good faith or else be liable for damages. Liability of partner in case of eviction Even a partnership for a fixed term may be The partner is bound in the same cases and in the terminated by the express will of any partner before the same manner as the vendor is bound with respect to the time mentioned. There is no such thing as an vendee with regard to specific and determinate things indissoluble partnership. which he may have contributed. This matter is governed by the law on sales. Continuation of partnership for an indefinite term Partnership for a term impliedly fixed An Liability of partner for fruits of property in case of agreement may evidence an understanding that the delay relation should continue until the accomplishment of a No demand is necessary to put the partner at fault. particular undertaking or certain things have been done The injury to the partnership is constant. or have taken place. Partnership with mere expectation that Liability of partner for failure to perform service business will be profitable A hope that the stipulated partnership earnings would pay for all the necessary Partner generally not liable Unless there is a expenses does not establish even by implication a fixed special agreement to that effect, the partners are not term or particular undertaking. Mere expectation that a entitled to charge each other, or the partnership, for business would be successful is not sufficient to create a their services in the firm business. partnership for a term. Exception The general rule that partners are not entitled to compensation for their services is inapplicable Art. 1786. Every partner is a debtor of the where the reason of it fails. partnership for whatever he may have promised to If a partner neglects or refuses, without reasonable contribute thereto. cause, to render the service which he agreed to perform He shall also be bound for warranty in case of by reason of which the partnership suffered loss, he eviction with regard to specific and determinate should be responsible for this breach. things which he may have contributed to the If the partner is compelled to make good the loss, partnership, in the same cases and in the same each member of the firm, including himself, will receive

manner as the vendor is bound with respect to the his proportion of the amount in the distribution of assets vendee. He shall also be liable for the fruits this cannot be considered compensation for services thereof from the time they should have been rendered. The proper measure of damages in such case delivered, without the need of any demand. is the value of services wrongfully withheld. Obligations with respect to contribution of Art. 1787. When the capital or a part thereof property which a partner is bound to contribute consists of goods, their appraisal must be made in the manner prescribed in the contract of partnership, and in
Helen C. Arevalo 13 Section 3D

Finals Reviewer PARTNERSHIP 1st Sem; 2003


the absence of stipulation, it shall be made by partnership money or property received by him for a experts chosen by the partners, and according to specific purpose of the partnership. current prices, the subsequent changes thereof Where there is mere failure to return No being for the account of the partnership. estafa. Remedy: civil action for liquidation of the partnership and a levy of its assets. Appraisal of goods or property contributed Appraisal is necessary to determine how much has Art. 1789. An industrial partner cannot engage been contributed by the partners. In the absence of in business for himself unless the partnership stipulation, the share of each partner in the profits or expressly permits him to do so; and if he should do losses is in proportion to what he may have contributed. so, the capitalist partners may either exclude him The appraisal is made: from the firm or avail themselves of the benefits 1.) In manner prescribed by contract of partnership; which he may have obtained in violation of this 2.) If no stipulation, by experts chosen by the provision, with a right to damages in either case. partners and according to current prices. After the goods have been contributed, the Obligations of industrial partner partnership bears the risk or gets the benefits of An industrial partner is one who contributes his subsequent changes of value. industry, labor or services to the partnership. He is In the case of immovable property, the appraisal is considered the owner of his services, which is his made in the inventory of said property; otherwise it may contribution to the common fund. be made as provided in art. 1787. Unless the contrary is stipulated, he becomes a debtor of the partnership for his work or services from Art. 1788. A partner who has undertaken to the moment the partnership relation begins. In effect, contribute a sum of money and fails to do so the partnership acquires an exclusive right to avail itself becomes a debtor for the interest and damages of his industry. Consequently, if he engages in business from the time he should have complied with his for himself, such act is considered prejudicial to the obligation. interest of the other partners. The same rule applies to any amount he may Action for specific performance not available against have taken from the partnership coffers, and his him involuntary servitude. liability shall begin from the time he converted the amount to his own use. Prohibition against engaging in business As regards an industrial partner Absolute Obligations with respect to contribution of money prohibition: any kind of business. and money converted to personal use As regards capitalist partners Prohibition 1.) To contribute on the date due the amount he extends only to any operation which is of the same kind has undertaken to contribute; of business in which the partnership is engaged. 2.) To reimburse any amount he may have taken from the partnership coffers and converted to Remedies where industrial partner engages in his own use; business 3.) To pay the agreed or legal interest, if he fails to The capitalist partners have the right either to: pay his contribution on time or in case he takes 1.) Exclude him from the firm; or any amount from the common fund and 2.) Avail themselves of the benefits which he may converts it to his own use; have obtained. 4.) To indemnify the partnership for the damages In either case, they have a right to damages. caused to it by the delay in the contribution or for the conversion of any sum for his personal Art. 1790. Unless there is a stipulation to the benefit. contrary, the partners shall contribute equal shares to the capital of the partnership. Liability of guilty partner for interest and damages The guilty partner is liable for interest and damages Extent of contribution to partnership capital not from the time judicial or extra judicial demand is Partners can stipulate contribution of unequal funds made but from the time he should have complied with to the common fund, but in the absence of such his obligation or from the time he converted the amount stipulation, the presumption is that their contribution to his own use. Unless otherwise stipulated, obligation to shall be in equal shares. contribute arises from the commencement of the Obviously, this does not apply to an industrial

partner partnership (perfection of the contract). unless he also contributes capital. Liability of partner for failure to return partnership Art. 1791. If there is no agreement to the money received contrary, in case of an imminent loss of the Where fraudulent misappropriation committed business of the partnership, any partner who Partner is guilty of estafa if he misappropriates refuses to contribute an additional share to the capital, except an industrial partner, to save the
Helen C. Arevalo 14 Section 3D

Finals Reviewer PARTNERSHIP 1st Sem; 2003


venture, shall be obliged to sell his interest to the Does not apply to partner not authorized to manage. other partners. Where manner of mgt not agreed upon and all partners participate in mgt, every partner considered managing Obligation of capitalist partner to contribute partner. additional capital Right of debtor to application of payment General rule: Capitalist partner not bound to Debtor given right to prefer payment of credit of partner contribute more than what he agreed to. if it should be more onerous to him. Except: In case of imminent loss of the business, and there is no agreement to the contrary, he is under Art. 1793. A partner who has received, in whole obligation to contribute an additional share to save the or in part, his share of a partnership, when the venture. If he refuses to contribute, he shall be obliged other partners have not collected theirs, shall be to sell his interest to the other partners. obliged, if the debtor should thereafter become Requisites for application of rule insolvent, to bring to the partnership capital what 1.) Imminent loss of the business of the he received even though he may have given partnership; receipt for his share only. 2.) Majority of capitalist partners are of the opinion that an additional contribution to the common Obligation of partner who receives share of fund would save the business; partnership credit 3.) Capitalist partner refuses deliberately (not coz Requisites for application of rule he aint got no money) to contribute an 1.) A partner has received, in whole or in part, his additional share to the capital; share of the partnership credit; 4.) There is no agreement that even in case of an 2.) The other partners have not collected their imminent loss of the business the partners are shares; not obliged to contribute. 3.) The partnership debtor has become insolvent. The industrial partner is exempt. Having contributed Reason for imposing obligation to return the his entire industry, he can do nothing further. debt becomes a bad debt. It would be unjust for that Reason for the sanction Refusal of partner to one partner not to share in the loss. Provision is based contribute additional share reflects lack of interest in the on community of interest among the partners. continuance of the partnership. Unjust for him to reap benefits when he doesnt also help. Credit collected after dissolution of the partnership Art. 1792. If a partner authorized to manage Does the obligation refer only to that collected during collects a demandable sum, which was owed to the existence of the partnership or does it also refer to him in his own name, from a person who owed the that collected after dissolution? partnership another sum also demandable, the Some commentators answer this question in the sum thus collected shall be applied to the two affirmative basing their answer in the community and credits in proportion to their amounts, even equality which ought to exist among the partners. though he may have given a receipt for his own Manresa and Ricci believed otherwise. Reasons: credit only; but should he have given it for the 1.) It would not be just that he who diligently account of the partnership credit, the amount shall collected his quota should suffer the be fully applied to the latter. consequence of the negligence of his associates. The provisions of this article are understood to 2.) Upon dissolution, the tie that unites the be without prejudice to the right granted to the partnership ceases. Thus, the reason for the debtor by article 1252, but only if the personal obligation disappears. credit of the partner should be more onerous to him. Art. 1794. Every partner is responsible to the partnership for damages suffered by it through his Obligation of managing partner who collects debt fault, and he cannot compensate them with the Requisites for application of rule profits and benefits which he may have earned for 1.) There exists at least two debts, one where the the partnership by his industry. However, the collecting partner is creditor, and the other, courts may equitably lessen this responsibility if where the partnership is creditor; through the partners extraordinary efforts in 2.) Both debts are demandable; other activities of the partnership, unusual profits 3.) The partner who collects is authorized to have been realized. manage and actually manages the partnership. Reason for applying payment to partnership Obligation of partner for damages to partnership

credit The law safeguards the interests of the This article follows the general rule in contracts that partnership by preventing the possibility of their being any person guilty of negligence or fault in the fulfillment subordinated by the managing partner to his own of his obligation shall be liable for damages. The interest to the prejudice of the other partners. partners fault, however, must be determined in
Helen C. Arevalo 15 Section 3D

Finals Reviewer PARTNERSHIP 1st Sem; 2003


accordance with the nature of the obligation and the of loss borne by partner since he remains owner. He is circumstances of the person, time and place. debtor to partnership for what he promised to contribute. If loss is due to any of the partners, they Compensation of damages with profits earned for shall be liable for damages. partnership by guilty partner Damages not generally subject to set-off As a Art. 1796. The partnership shall be responsible general rule, the damages caused by a partner to the to every partner for the amounts he may have partnership cannot be offset by the profits or benefits disbursed on behalf of the partnership and for the which he may have earned for the partnership by his corresponding interest, from the time the industry. expenses are made; it shall also answer to each Partner has obligation to secure benefits for partner for the obligations he may have contracted partnership. Profits which he may have earned are the in good faith in the interest of the partnership partnerships as a matter of law or right. business, and for risks in consequence of its He also has the obligation to exercise diligence. management. Partner is debtor to partnership for his industry, and at the same time obliged to repair injury which he might Responsibility of the partnership to the partners have caused through his fault. In the absence of stipulation to the contrary, every Exception If unusual profits are realized through partner is an agent of the partnership for the purpose of the extraordinary efforts of the partner at fault, the its business. Hence, the partnership has the obligation courts may equitably mitigate or lessen his liability for to: damages. This rule rests on equity. 1.) Refund amounts disbursed by the partner in behalf of the partnership plus interest from time Art. 1795. The risk of specific and determinate expenses made; things, which are not fungible, contributed to the 2.) Answer for obligations the partner may have partnership so that only their use and fruits may contracted in good faith in the interest of the be for the common benefit, shall be borne by the partnership business; partner who owns them. 3.) Answer for risks in consequence of its If the things contributed are fungible, or cannot management. be kept without deteriorating, or if they were Being a mere agent, the partner is not personally contributed to be sold, the risk shall be borne by liable as long as he is free from fault and acted within the partnership. In the absence of stipulation, the the scope of his authority. But unlike an ordinary agent, risks of things brought and appraised in the he is not give the right of retention if he is not inventory, shall also be borne by the partnership, reimbursed or indemnified. and in such case the claim shall be limited to the value at which they were appraised. Art. 1797. The losses and profits shall be distributed in conformity with the agreement. If Risk of loss of things contributed only the share of each partner in the profits has 1.) Specific and determinate things which are not been agreed upon, the share of each in the losses fungible where only the use is contributed Risk shall be in the same proportion. of loss is borne by the partner because he In the absence of stipulation, the share of each remains the owner of the things. partner in the profits and losses shall be in 2.) Specific and determinate things the ownership of proportion to what he may have contributed, but which is transferred to the partnership Risk of the industrial partner shall not be liable for losses. loss for the account of partnership as owner. As for the profits, the industrial partner shall 3.) Fungible things or things which cannot be kept receive such share as may be just and equitable without deteriorating even if they are under the circumstances. If besides his services he contributed only for the use of the partnership has contributed capital, he shall also receive a Risk of loss borne by partnership. Ownership share in the profits in proportion to his capital. was being transferred since use is impossible without the things being consumed or impaired. Rules for distribution of profits and losses 4.) Things contributed to be sold Partnership Distribution of profits bears risk of loss. Partnership intended to be 1.) The partners share the profits according to their owner or it could not effect the sale. agreement subject to art. 1799. 5.) Things brought and appraised in the inventory- 2.) If there is no such agreement:

Partnership bears risk of loss because intention a.) The share of each capitalist partner shall of parties was to contribute to the partnership be in proportion to his capital the price of the things contributed with an contribution. appraisal in the inventory. There is thus an b.) Industrial partner shall receive such implied sale making the partnership owner. share, which must be satisfied first The above presuppose that the things have been before the capitalist partners shall divide delivered actually or constructively. Before delivery, risk
Helen C. Arevalo 16 Section 3D

Finals Reviewer PARTNERSHIP 1st Sem; 2003


the profits, as may be just and equitable and profits or losses shall be apportioned as if there under the circumstances. It is not fixed. were no stipulation. A partner is entitled to receive only his share of the Stipulation, a factor to show no partnership profits actually realized by the venture. Even when exists such a stipulation may be a factor in assurance was made by a partner that they would earn determining that no partnership exists. a huge amt of profits, in the absence of fraud, the Where person excluded not intended by parties others cannot claim a right to recover profits promised to become a partner Stipulation valid. However, if where the business was highly speculative and turned held himself out to be partner, liable. out to be a failure. Where person excluded from losses is industrial Distribution of losses partner Since law itself excludes him from losses, 1.) According to agreement. stipulation exempting him from such naturally valid. 2.) No such agreement, but contract provides for Where stipulation provides for unequal shares share in profits, share in losses in accordance This is fine as long as the inequality is not so gross that with profit-sharing ratio, but industrial partner it amounts to exclusion. not liable for losses. 3.) No profit-sharing stipulated, losses in proportion Stipulation exempting a partner from losses to capital contributions, but purely industrial should be allowed partner not liable for losses. If a person can make a gift to another, there is no sound reason why a person cannot also bear all the Art. 1798. If the partners have agreed to intrust losses that a partnership may suffer, in order to exempt to a third person the designation of the share of his co-partners from sharing in the said losses. each one in the profits and losses, such As far as third persons concerned, such stipulation designation may be impugned only when it is may properly be declared void. manifestly inequitable. In no case may a partner who has begun to execute the decision of the third Art. 1800. The partner who has been appointed person, or who has not impugned the same within manager in the articles of partnership may a period of three months from the time he had execute all acts of administration despite the knowledge thereof, complain of such decision. opposition of his partners, unless he should act in The designation of losses and profits cannot be bad faith; and his power is irrevocable without just intrusted to one of the partners. or lawful cause. The vote of the partners representing the controlling interest shall be Designation by a third person of share in profits necessary for such revocation of power. and losses A power granted after the partnership has been Delegation to a third person Designation of constituted may be revoked at any time. share in profits and losses may be delegated to a 3rd person by common consent. Rights and obligations with respect to Prohibition in 2nd paragraph necessary to guarantee management utmost impartiality in distribution. Each partner has a right to an equal voice in the Binding force of designation by 3rd person conduct of the partnership business. This right is not Generally binding unless manifestly inequitable. Even dependent on the amount or size of the partners capital then, a partner who has begun to execute the decision contribution. of the 3rd person or who fails to impugn the same within Appointment as manager in the arts of 3 months from the time he had knowledge of it can no partnership Partner appointed in arts of partnership longer complain. He is estopped, deemed to have given may execute all acts of administration notwithstanding consent or ratification. the opposition of the other partners, unless he should Reason for 3 months: forestall any paralyzation in act in bad faith. His power is revocable only upon just operations of partnership. and lawful cause and upon the vote of the partners representing the controlling interest. Art. 1799. A stipulation which excludes one or Reason: revocation represents change in terms of more partners from any share in the profits or contract. losses is void. In case of mismanagement: Usual remedies allowed by law including dissolution. Stipulation excluding a partner from any share in Appointment as manager after the constitution

profits or losses of the partnership Appointment may be revoked at Stipulation generally void, but partnership any time for any cause whatsoever. subsists Partnership must exist for common benefit Reason: revocation not founded on a change of will and interest of the partners. Hence, such a stipulation on the part of the partners. Appointment not condition contravenes the very purpose of partnership contract of contract. It is merely a simple contract of agency profit-sharing among partners. However, although which may be revoked at any time. It is believed that stipulation void, partnership otherwise valid, subsists the vote for revocation must also represent the controlling interest.
Helen C. Arevalo 17 Section 3D

Finals Reviewer PARTNERSHIP 1st Sem; 2003


1.) If one or more of the managing partners shall Scope of power of a managing partner oppose the acts of the others, then the decision General rule: partner appointed as manager has all of the majority of the managing partners shall the powers of a general agent as well as all the prevail. Right to oppose can be exercised only by incidental powers necessary to carry out the object of those entrusted with mgt. the partnership in the transaction of its business. 2.) In case of tie, matter shall be decided by the Exception: when powers of mgr specifically restricted. vote of the partners owning the controlling A managing partner may not bind the partnership by interest. a contract wholly foreign to its business. REQUISITES FOR APPLICATION OF RULE 1.) Two or more partners have been appointed as Compensation for services rendered managers; Partner generally not entitled to compensation 2.) There is no specification of their respective In the absence of an agreement to the contrary, each duties; member of the partnership assumes the duty to give his 3.) There is no stipulation that one of them shall not time, attention, and skill to the mgt of its affairs, so far, act without the consent of all the others. at least, as may be reasonably necessary to the success of the common enterprise; and for this service a share Art. 1802. In case it should have been of the profits is his only compensation. stipulated that none of the managing partners In managing partnership affairs, a partner is shall act without the consent of the others, the practically taking care of his own interest or managing concurrence of all shall be necessary for validity of his own business. the acts, and the absence or disability of any one In the absence of any prohibition in the arts. of of them cannot be alleged, unless there is partnership for the payment of salaries to general imminent danger of grave or irreparable injury to partners, there is nothing to prevent the partners to the partnership. enter into a collateral verbal agreement to that effect. Exceptions In proper cases, the law may imply a When unanimity of action stipulated contract for compensation: Concurrence necessary for validity of acts The 1.) A partner engaged by his co-partners to perform partners may stipulate that none of the managing services not required of him in fulfillment of the partners shall act without the consent of the others. In duties and in capacity other than that of a such a case, the unanimous consent of all the managing partner. partners shall be necessary for the validity of their acts. 2.) When there is extraordinary neglect on the part This consent is so indispensable that neither absence of one partner to perform his duties, imposing nor disability of any one of them may be alleged as entire burden on remaining partner. excuse to dispense with requirement. 3.) One partner may employ the other to do work Exception: When there is imminent danger of grave for him outside of and independent of the co-or irreparable injury to the partnership then a partner partnership. may act alone without consent of partner who is absent 4.) Partners exempted by terms of partnership from or under disability. rendering services may demand pay for services Consent of managing partners not necessary in rendered. routine transactions The requirement of written 5.) Where one partner is entrusted w/ mgt and authority refers evidently to formal and unusual written devotes his whole time and devotion at the contracts. instance of the other partners who are attending to their individual business and giving no time or Art. 1803. When the manner of management has not attention to the partnership business. been agreed upon, the following rules shall be observed: Art. 1801. If two or more partners have been 1.) All the partners shall be considered intrusted with the management of the partnership agents and whatever any one of them without specification of their respective duties, or may do alone shall bind the partnership, without stipulation that one of them shall not act without prejudice to the provisions of without the consent of all the others, each one article 1801. may separately execute all acts of administration, 2.) None of the partners may, without the but if any one of them should oppose the acts of consent of the others, make any

the others, the decision of the majority shall important alteration in the immovable prevail. In case of tie, the matter shall be decided property of the partnership, even if it may by the partners owning the controlling interest. be useful to the partnership. But if the refusal of consent by the other partners is Where respective duties of two or more managing manifestly prejudicial to the interest of partners not specified the partnership, the courts intervention Each one may separately perform acts of may be sought. administration
Helen C. Arevalo 18 Section 3D

Finals Reviewer PARTNERSHIP 1st Sem; 2003


Rules when manner of management has not been agreed upon Keeping of partnership books All partners considered managers and agents Partner with duty to keep partnership books All partners shall have equal rights in the mgt and The duty to keep true and correct books showing the conduct of partnership affairs. All of them shall be firms accounts, such books being at all times open to considered mgrs and agents and whatever any one of inspection of all members of the firm, primarily rests on them may do alone shall bind the partnership. If there is the managing or active partner. It is presumed that the timely opposition, however, the matter shall be decided partners have knowledge of the contents of the by majority vote. In case of tie, vote of partners partnership books and that said books state accurately representing controlling interest. the state of accounts, but errors can be corrected. Unanimous consent required for alteration of Rights with respect to partnership books immovable property The consent need not be Books should be kept at the principal place of business express. It may be presumed from the fact of knowledge as each partner has the right to free access to them and of the alteration without interposing any objection. to inspect or copy any of them at any reasonable time, Prohibition only applies to immovable property even after dissolution. because of the greater importance of this kind of Inspection rights not absolute, can be restrained property, and the alteration thereof must be important. from using info for other than partnership purpose. This would be an act of strict dominion. Access to partnership books Rights can be If refusal to give consent is manifestly prejudicial to exercised at any reasonable hour. This means the interest of the partnership, court intervention may reasonable hours on business days throughout the year be sought. Consent may be presumed from silence (lack and not merely during some arbitrary period of a few of opposition despite knowledge). days chosen by the managing partners. If alteration is necessary for preservation of the property, consent of the other partners not required. Art. 1806. Partners shall render on demand true and full information of all things affecting the Art. 1804. Every partner may associate another partnership to any partner or the legal person with him in his share, but the associate representative of any deceased partner or of any shall not be admitted into the partnership without partner under legal disability. the consent of all the other partners, even if the partner having an associate should be a manager. Duty to render information There must be no concealment between partners in Contract of subpartnership all matters affecting the partnership. Info must be used Nature The partnership formed between a only for partnership purpose. member of a partnership and a 3rd person for a division Not just on demand but partner also has duty of of the profits coming to him from the partnership voluntary disclosure. But duty to render info does not enterprise is termed subpartnership. arise with respect to matters appearing in partnership It is a partnership within a partnership and is distinct books since each partner has the right to inspect those. and separate from the main or principal partnership. Good faith not only requires that a partner should not Right of person associated with partners share make a false statement but also that he should abstain Subpartnership agreements do not affect the from any false concealment. composition, existence, or operations of the firm. The subpartners are partners inter se, but in the absence of Art. 1807. Every partner must account to the

the mutual assent of all the parties, a subpartner does partnership for any benefit, and hold as trustee for not become a member of the partnership, even if the it any profits derived by him without the consent other partners know about the agreement. of the other partners from any transaction Not being a member of the partnership, he does not connected with the formation, conduct, or acquire the rights of a partner nor is he liable for its liquidation of the partnership or from any use by debts. him of its property. Reason for the rule Partnership is based on mutual trust and confidence among the partners. Partner accountable as fiduciary Inclusion of new partner would be a modification of the The relation between the partners is essentially original contract of partnership requiring unanimous fiduciary involving trust and confidence, each partner consent of all the partners. Prohibition applies even if being considered in law, as he is, in fact, the confidential person associated is already a partner. agent of the others. The duties of a partner are analogous to those of a trustee. Art. 1805. The partnership books shall be kept, Duty to act for common benefit Cannot use and subject to any agreement between the partners, at apply exclusively to own individual benefit partnership the principal place of business of the partnership, assets or results of knowledge and info gained in and every partner shall at any reasonable hour character of partner. have access to and may inspect and copy any of Managing partners particularly owe a fiduciary duty them. to inactive partners.
Helen C. Arevalo 19 Section 3D

Finals Reviewer PARTNERSHIP 1st Sem; 2003


Duty begins during formation of partnership engaged and which is competitive w/ said business. Principle of good faith applies not only during Violation obligation to bring to common fund any partnership but during the negotiations leading to the profits derived and in case of losses, he shall bear them formation of the partnership. alone. Partners, however, by stipulation may permit it. Also, a person who agreed w/ another to form a The law permits him to carry on a business not partnership has the obligation to account for connected or competing with that of the partnership. commissions and discounts received in acquiring Law is silent on whether he can engage in same line property for the future partnership. of business for the account of another. Prohibition still Duty continues even after dissolution of applies because of fiduciary position imposing duties of partnership Duty of partner to act w/ utmost good utmost good faith. He may not carry on any other faith towards his co-partners continues throughout the business in rivalry w/ the partnership. entire life of the partnership even after dissolution for Reason for prohibition Fiduciary nature of whatever reason or whatever means, until the relationship imposes obligation of utmost good faith. relationship is terminated, i.e. the winding up of Rule prevents use of info obtained in course of partnership affairs is completed. transaction of partnership business or by reason of Duty to account for secret and similar profits connection w/ firm regarding business secrets and The duty of a partner to account as a fiduciary operates clientele of firm to its prejudice. to prevent from making a secret profit out of the operation of the partnership and from carrying on the Art. 1809. Any partner shall have the right to a business for his private advantage or a business in formal account as to partnership affairs: competition w/ the firm w/o consent of other partners. 1.) If he is wrongfully excluded from the Violation may be ground for dissolution. partnership business or possession of its Duty to account for earnings accruing even property by his co-partners; after termination of partnership If a partner uses 2.) If the right exists under the terms of any info obtained by him from the partnership for his own agreement; account w/o the consent of the other partners, he is 3.) As provided by article 1807; liable to account for any benefit he might obtain. 4.) Whenever other circumstances render it Duty to make full disclosure of information just and reasonable. belonging to partnership A partner is also subject to the fiduciary duty of undivided loyalty and complete Right of partner to a formal account disclosure of info of all things affecting the partnership. General rule During existence of partnership, a By information is meant information which can be partner is not entitled to a formal account of partnership used for the purposes of the partnership. Info cannot be affairs. Reason: rights of partner amply protected in arts used for a partners private gain even if after 1805 and 1806. Also, it would cause much termination. inconvenience and unnecessary waste of time. Duty not to acquire interest or right adverse to Exceptions In the special and unusual situations partnership If partner does, he holds it in trust for enumerated under art. 1809. the benefit of the partnership and must account to the Right of partner to demand an accounting w/o firm for the profits of the transaction, unless it appears bringing about dissolution is a necessary corollary to that the others consented (must be informed consent, right to share in profits. A formal account is a necessary i.e. with knowledge of facts necessary to the giving of an incident to the dissolution of the partnership. intelligent consent.) Prescriptive period Right to demand accounting exists as long as partnership lasts. Prescription only Art. 1808. The capitalist partners cannot begins to run upon dissolution when final accounting is engage for their own account in any operation done. which is of the kind of business in which the partnership is engaged, unless there is a stipulation to the contrary. Any capitalist partner violating this prohibition SECTION 2. PROPERTY RIGHTS OF A PARTNER shall bring to the common funds any profits accruing to him from his transactions, and shall Art. 1810. The property rights of a partner are: His rights in specific partnership property;

personally bear all the losses. 1.) His interest in the partnership; and 2.) His right to participate in the management. Prohibition against partner engaging in business 3.) Prohibition relative Prohibition against capitalist partner to engage in business is relative, unlike the Extent of property rights of a partner industrial partner who is absolutely prohibited from Principal rights Rights in specific partnership property; engaging in any business for himself. 1.) Interest in partnership; Capitalist partner is only prohibited from engaging for 2.) Right to participate in management. his own account in any operation which is the same as 3.) or similar to the business in which the partnership is
Helen C. Arevalo 20 Section 3D

Finals Reviewer PARTNERSHIP 1st Sem; 2003


RELATED RIGHTS The incidents of this co-ownership are such 1.) Right to reimbursement for amounts advanced that: to partnership and to indemnification for risks in 1.) A partner, subject to the provisions of this consequence of management (art. 1796). Title and to any agreement between the 2.) Right of access and inspection of partnership partners, has an equal right with his books (art. 1805). partners to possess specific partnership 3.) Right to true and full information of all things property for partnership purposes; but he affecting partnership (art. 1806). has no right to possess such property for 4.) Right to formal account of partnership affairs any other purpose without the consent of under certain circumstances (art. 1809). his partners; 5.) Right to have partnership dissolved also under 2.) A partners right in specific partnership certain conditions (arts. 1830-1831). property is not assignable except in connection with the assignment of rights of Partnership property and partnership capital all the partners in the same property; distinguished 3.) A partners right in specific partnership Partnership Partnership property is not subject to attachment or property capital execution, except on a claim against the Changes Variable: its value Constant: it partnership. When partnership property is in value may vary from day to remains unchanged attached for a partnership debt the day w/ changes in as the amount is partners, or any of them, or the market value. fixed by agreement representatives of a deceased partner, of the partners, and cannot claim any right under the is not affected by homestead or exemption laws; fluctuations in the 4.) A partners right in specific partnership value of the property is not subject to legal support partnership under art. 291. property, although it may be increased Nature of a partners right in specific partnership and decreased by property unanimous consent Art. 1811 contemplates tangible property but not of the partners. intangible things. Assets Includes not only the The aggregate of A partner is a co-owner w/ his partners of specific included original capital the individual partnership property, but the rules on co-ownership do contributions, but all contributions made not necessarily apply. The legal incidents of this tenancy property subsequently by the partners in in partnership are distinctively characteristic of the acquired on account establishing or partnership relation. They are as follows: of the partnership or continuing the Equal right of possession Ordinarily, a partner w/ partnership funds, partnership. has an equal right to possess specific partnership including partnership property for partnership purposes. None of the partners name and goodwill. can possess and use the specific partnership property other than for partnership purposes w/o the consent of Ownership of certain property the other partners. Property used by the partnership Where there Should any of them use the property for his own is no express agreement that property used by a benefit, he must account, like a stranger, to the others partnership constitutes partnership property, such use for the profits derived therefrom or the value of his does not make it partnership property, and whether it is wrongful possession or occupation. A partner wrongfully so depends on the intention of the parties, w/c may be excluded from possession of partnership property by a shown by proving an express agreement or acts of co-partner has a right to formal account and may even particular conduct. apply for a judicial decree of dissolution. The intent of the parties is the controlling factor. On the death of a partner, his right in specific Property acquired by a partner w/ partnership partnership property vests in the surviving partners. funds Unless a contrary intention appears, property By agreement, the right to possess specific

acquired by a partner in his own name w/ partnership partnership property may be surrendered. In the funds is partnership property. But if the property was absence of special agreement, however, neither partner acquired after dissolution but before the winding up of separately owns, or has the exclusive right of possession the partnership affairs, it would be his separate property of any partnership property or any proportional part but he would be liable to account to the partnership for thereof. Each has dominion over the entire partnership the funds used in its acquisition. property. The possession of partnership property by one Art. 1811. A partner is co-owner with his partner is the possession of all until his possession partners of specific partnership property. becomes adverse. A partner cannot initiate title by
Helen C. Arevalo 21 Section 3D

Finals Reviewer PARTNERSHIP 1st Sem; 2003


adverse possession until and unless he makes an undistributed profits during the life of the partnership as adverse claim. a going concern and his share in the undistributed Right not assignable A partner cannot separately surplus after its dissolution. assign his right to specific partnership property but all of Profits: the excess of returns over expenditure in a them can assign their rights in the same property. transaction or series of transactions; or the net income of the Reasons for non-assignability:partnership for a given period of time. 1.) It prevents interference by outsiders in Surplus: the assets of the partnership after partnership partnership affairs; debts and liabilities are paid and settled and the rights of the 2.) It protects the right of other partners and partners among themselves are adjusted. It is the excess of partnership creditors to have partnership assets assethe difference represents the extent of the loss. applied to firm debts; Extent of the partners interest The interest in 3.) It is often impossible to determine the extent of surplus alone w/c remains after the firms debts have a partners beneficial interest in a particular been paid and the equities between the partner and his partnership asset. co-partners have been adjusted and the partners share Reason for impossibility: Each partner, having a has been ascertained and set apart. beneficial interest in the partnership property considered Partners interest not a debt due from as a whole, has a beneficial interest in each part. partnership A partner is not a creditor of the Where, however, none of the above reasons apply, partnership for the amount of his share. an authorized assignment by a partner of his right in specific partnership property is void, but it may be Art. 1813. A conveyance by a partner of his regarded as a valid assignment of the partners interest whole interest in the partnership does not of itself in the partnership. dissolve the partnership, or, against the other The law allows a retiring partner to assign his rights partners in the absence of agreement, entitle the in partnership property to the partner(s) continuing the assignee, during the continuance of the business. partnership, to interfere in the management or Right limited to share of what remains after administration of the partnership business or partnership debts have been paid Strictly affairs, or to require any information or account of speaking, no particular partnership property or any partnership transactions, or to inspect the specific or an aliquot part thereof can be considered the partnership books; but it merely entitles the separate or individual property of any partner. The assignee to receive in accordance with his contract whole of partnership property belongs to the partnership the profits to which the assigning partner would considered as a juridical person, and a partner has no otherwise be entitled. However, in case of fraud in interest in it but his share of what remains after all the management of the partnership, the assignee partnership debts are paid. may avail himself of the usual remedies. Consequently, specific partnership property is not In case of dissolution of the partnership, the subject to attachment, execution, garnishment, or assignee is entitled to receive his assignors injunction, w/o the consent of all the partners except on interest and may require an account from the date a claim against the partnership. only of the last account agreed to by all the For the same reason that the property belongs to the partners. partnership, the partners cannot claim any right under the homestead or exemption laws when it is attached for Effect of assignment of partners whole interest in partnership debts. But a partners interest in the partnership partnership itself may be levied upon by a judgment A partners right in specific partnership property is creditor because it is actually his property, by means of not assignable but he may assign his interest in the a charging order. partnership to any of his co-partners or to a 3rd person The right of the partners to specific partnership irrespective of the consent of the other partners, in the property is not subject to legal support since the absence of agreement to the contrary. property belongs to the partnership and not to the Rights withheld from assignee partners. But their interest in the partnership is. 1.) To interfere in the management;
ts over liabilities. If the liabilities are more than the assets,

The method of reaching a judgment debtors interest 2.) To require any information or account; in partnership property is specifically set forth in art. 3.) To inspect any of the partnership books. 1814. Nature of partnership relation The legal effect of such a conveyance is the same as that of a partner Art. 1812. A partners interest in the associating another in his share or interest. Remember partnership is his share of the profits and surplus. delectus personarum. No-one can be compelled to be partners w/ someone else. The assignment does not EMPHASIZE! divest the assignor of his status and rights as a partner nor operate as a dissolution. The law, however, provides Nature of partners interest in the partnership the non-assigning partners w/ a ground for dissolving Share of the profits and surplus The partners the partnership if they so desire. interest in the partnership consists of his share in the
Helen C. Arevalo 22 Section 3D

Finals Reviewer PARTNERSHIP 1st Sem; 2003


Remedy of other partners Remedies of separate judgment creditor of a Dissolution of partnership not intended Many partner partnership agreements are made merely as security for Application for a charging order after loans, the assigning partner never intending to destroy securing judgment on his credit While a separate the partnership relation. If the assigning partner creditor of a partner cannot attach or levy upon specific neglects his duties after assignment, the other partners partnership property for the satisfaction of his credit may dissolve the partnership under art. 1830(1,c). because partnership assets are reserved for partnership Dissolution of partnership intended A partners creditors, he can secure a judgment on his credit and conveyance of his interest in the partnership operates as then apply to the proper court for a charging order, a dissolution of the partnership only when it is clear that subjecting the interest of the debtor partner in the the parties contemplated and intended the entire partnership w/ the payment of the unsatisfied amount of withdrawal from the partnership of such partner and the such judgment w/ interest thereon w/ the least termination of the partnership as between the partners. interference w/ the partnership business and the rights of the other partners. By virtue of the charging order, Rights of assignee of partners interest any amount or portion thereof w/c the partnership 1.) To receive in accordance w/ his contract the would otherwise pay to the debtor-partner should profits accruing to the assigning partner; instead be given to the judgment creditor. 2.) To avail himself of the usual remedies provided This remedy, however, is w/o prejudice to the by law in the event of fraud in the management; preferred rights of partnership creditors whose claims 3.) To receive the assignors interest in case of should be satisfied first. dissolution; Availability of other remedies Art. 1814 have 4.) To require an account of partnership affairs, but made this an exclusive remedy so that a writ of only in case the partnership is dissolved, and execution will not be proper. However, if the judgment such account shall cover the period from the date debt remains unsatisfied, the court may resort to other only of the last account agreed to by all partners. courses of action notwithstanding the issuance of the The purchaser of a partners interest may apply to charging order. the court for dissolution after the termination of the specified term or undertaking or at any time if the Redemption or purchase of interest charged partnership is one at will. Redemptioner The interest of the debtor-partner so charged may be redeemed or purchased w/ the Art. 1814. Without prejudice to the preferred separate property of any one or more of the partners, or rights of partnership creditors under art. 1827, on w/ partnership property but w/ the consent of all the due application to a competent court by any partners whose interests are not so charged or sold. judgment creditor of a partner, the court which Redemption price The value of the partners entered the judgment, or any other court, may interest in the partnership has no bearing on the charge the interest of the debtor partner with redemption price w/c is likely to be lower since it will be payment of the unsatisfied amount of such dependent on the amount of the unsatisfied judgment judgment debt with interest thereon; and may debt. then or later appoint a receiver of his share of the Right of redeeming non-debtor partner The profits, and of any other money due or to fall due redeeming non-debtor partner does not acquire absolute to him in respect of the partnership, and make all ownership over the debtor-partners interest but holds it other orders, directions, accounts and inquiries in trust for him consistent w/ principles of fiduciary which the debtor partner might have made, or relationship. which circumstances of the case may require. The interest charged may be redeemed at any Rights of partner under exemption laws time before foreclosure, or in any case of a sale Under art. 1811, a partner cannot claim any right

being directed by the court, may be purchased under the homestead laws or exemption laws when without thereby causing a dissolution: specific partnership property is attached for partnership 1.) With separate property, by any one or debt. more of the partners; or W/ respect, however, to the partners interest in the 2.) With partnership property, by any one or partnership as distinguished from his interest in specific more of the partners with the consent of partnership property, the partner may avail himself of all the partners whose interest are not so the exemption laws after partnership debts have been charged or sold. paid. A partners interest or share in the partnership Nothing in this Title shall be held to deprive a property is really his property. partner of his right, if any, under the exemption laws, as regards his interest in the partnership. EMPHASIZE! SECTION 3 OBLIGATIONS OF THE PARTNERS WITH REGARD TO THIRD PERSONS

Helen C. Arevalo 23 Section 3D

Finals Reviewer PARTNERSHIP 1st Sem; 2003


Art. 1815. Every partnership shall operate EMPHASIZE! under a firm name, which may or may not include the name of one or more of the partners. Liability for contractual obligations of the Those who, not being members of the partnership partnership, include their names in the firm name, Partnership liability Partners are principals to the shall be subject to the liability of a partner. other partners and agents for them and the partnership. They are liable to 3rd persons who have dealt with one of Requirement of a firm name them in the same way that a principal is liable to 3rd Meaning of word firm: The name, title, or style persons who have dealt with an agent. The general rule under which a company transacts business; a is that a partner has the right to make all partners liable partnership pf two or more persons; a commercial for contracts he makes for the partnership in the name house. In its common acceptation, the term implies a and for the account of the partnership. partnership. The term is also used as synonymous with Individual liability A partner, however, may company, house, and concern. assume a separate undertaking in his name with a 3rd Importance of having a firm name A party to perform a partnership contract or make himself partnership must have a firm name under which it will solidarily liable on a partnership contract. In such case, operate. A firm name is necessary to distinguish the the partner is personally bound by his contract even if partnership which has a distinct and separate juridical only the partnership is shown to have derived benefits personality from the individuals composing the from it. partnership and from other partnerships and entites. Right of partners to choose firm name The Nature of individual liability of partners partners enjoy the utmost freedom in the selection of Pro rata Equally or jointly, not proportionately. Pro the partnership name. As a general rule, they may adopt rating is based on the number of partners and not on any firm name desired. the amount of their contributions to the common fund, Use of misleading name The partners cannot subject to adjustment among the partners. use a name that is identical or deceptively confusingly Subsidiary or secondary It is subsidiary or similar to that of any existing partnership or corporation secondary because the partners become personally or to any other name already protected by law or is liable only after all the partnership assets have been patently deceptive, confusing or contrary to existing exhausted. Thus, the partners are liable as guarantors in laws, as to mislead the public by passing itself off as favor of partnership creditors to the extent that the another partnership or corporation, or its goods or asets of the firm are not sufficient to meet its services as those of such other company. obligations. They may be joined as party defendants in Use of names of deceased partners Ruling of the same action against the partnership subject to their SC in Ozaeta, Romulo, etc. abandoned. Rule 3.02 of the right to prior exhaustion of partnership property. Code of Professional Responsibility provides that the Even the industrial partner who, ordinarily, is not continued use of the name of a deceased partner is liable for losses would have to pay but, of course, he can permissible provided that the firm indicates in all its recover the amount he has paid from the capitalist communications that said partner is deceased. partners unless there is an agreement to the contrary. Liability for inclusion of name in firm name Distinction between a liability and a loss Persons who, not being partners, include their names The inability of a partnership to pay debt to a 3rd in the firm name do not acquire the rights of a partner party at a particular time does not necessarily

mean that but shall be subject to the liability of a partner insofar as the partnership business, as a whole, has been operated 3rd persons without notice are concerned. Such persons at a loss. The partnership may have outstanding credits become partners by estoppel. which for the moment may be unavailable for the Art. 1815 does not cover the case of a limited partner payment of debts, but which eventually may be realized who allows his name to be included in the firm name, or upon and yield profits more sufficient to cover all losses. of a person continuing the business of a partnership Article 1816 refers to liabilities while Article 1767 after dissolution, who uses the name of the dissolved speaks of losses. There is, therefore, no conflict partnership or the name of a deceased partner as part between the two articles. thereof. The exemption of the industrial partner to pay losses relates exclusively to the settlement of the partnership Art. 1816. All partners, including industrial affairs among the partners themselves and has nothing ones, shall be liable pro rata with all their property to do with the liabilities of the partners to 3rd persons. and after all the partnership assets have been An industrial partner is not exempted from liability to 3rd exhausted, for the contracts which may be entered persons for the debts of the partnership. into in the name and for the account of the partnership. However, any partner may enter into No distinction between obligations and losses a separate obligation to perform a partnership During the existence of a partnership, the gains or contract. the losses are set off, and the difference is either in favor of or against the concern. As to the industrial
Helen C. Arevalo 24 Section 3D

Finals Reviewer PARTNERSHIP 1st Sem; 2003


partner, it is not a matter of striking a balance from time Power of partner as agent of partnership to time, but one of the final adjustment of assets and In the absence of an agreement to the contrary, all liabilities. As long as there is property belonging to the partners have equal rights in the management and partnership, obligations in favor of 3rd persons are conduct of the partnership business. covered by the primary and direct responsibility of the As among themselves When a partner performs partnership. an act within the scope of his actual, implied, or The question arises when the assets of the apparent authority, he is not only a principal as to partnership are exhausted and it becomes necessary to himself, but is also for all purposes, an agent as to his enforce the subsidiary liability of the private property of co-partners or to the partnership, considered as a the partners. In this case, such obligations constitute group. Principle of mutual agency. the extreme losses in the liquidation of the partnership. As to third persons Limitations upon the authority of any one of the partners are not binding Art. 1817. Any stipulation against the liability upon innocent 3rd persons dealing with the partnership laid down in the preceding article shall be void, who have the right to assume that every general partner except as among partners. has power to bind the partnership especially those partners acting with ostensible authority, by whatever is Stipulation against liability proper for the transaction in the ordinary and usual A stipulation among the partners contrary to the pro manner of the business of the partnership. rata and subsidiary liability expressly imposed by Article No duty to make inquiries as to acting 1816 is void and of no effect insofar as it affects the partners authority 3rd persons are not bound, in rights of 3rd persons. It is valid and enforceable only as entering a contract with any of the partners, to ascertain among the partners. whether or not the partner with whom the transaction is made has the consent of the other partners. His Art. 1818. Every partner is an agent of the knowledge is enough that he is contracting with a partnership for the purpose of its business, and partner. the act of every partner, including the execution in Presumption that acting partner has the partnership name of any instrument, for authority to bind partnership There is a general apparently carrying on in the usual way the presumption that each individual partner is an agent of business of the partnership of which he is a the firm and that he has authority to bind the firm in member, binds the partnership, unless the partner carrying on the partnership transactions. The so acting has in fact no authority to act for the presumption is sufficient to permit 3rd persons to hold partnership in the particular matter, and the the firm liable on transactions entered into by any one of person with whom he is dealing has knowledge of the members of the firm acting apparently in its behalf the fact that he has no such authority. and within the scope of his authority. An act of a partner which is not apparently for No right to assume that acting partner has carrying on of the business of the partnership in unlimited authority The apparent scope of the the usual way does not bind the partnership partners authority is the whole scope of the unless authorized by the other partners. partnerships customary business. However, 3rd parties Except when authorized by the other partners should not assume that a partner has unlimited or unless they have abandoned the business, one authority. Generally, a partner has no authority to do or more but less than all the partners have no the cats enumerated in the 3rd paragraph of Article authority to: 1818. When a 3rd party deals with a partner who has no 1.) Assign the partnership property in trust for express, implied, or apparent authority, the partnership creditors or on the assignees promise to is not liable for his acts unless the other partners ratify pay the debts of the partnership; his acts or are estopped from asserting the partners 2.) Dispose of the goodwill of the business; lack of authority. 3.) Do any other act which would make it impossible to carry on the ordinary Liability of partnership for acts of partners business of a partnership; The acts of a partner mentioned in Article 1818 may 4.) Confess a judgment; be grouped into three: 5.) Enter into a compromise concerning a 1.) Acts for apparently carrying on in the usual partnership claim or liability; way the business of the partnership (par. 1)

6.) Submit a partnership claim or liability to Every partner is an agent and may execute arbitration; such acts with binding affect on the partnership 7.) Renounce a claim of the partnership. even if he has in fact no authority unless the 3rd No act of a partner in contravention of a person has knowledge of such lack of authority. restriction on authority shall bind the partnership There are two requisites in order that the to persons having knowledge of the restriction. partnership will not be liable: a.) The partner so acting has in fact no EMPHASIZE! authority; and

Helen C. Arevalo 25 Section 3D

Finals Reviewer PARTNERSHIP 1st Sem; 2003


b.) The 3rd person knows that the acting record does not disclose the right of the partner has no authority. partnership, the partners in whose name the title Usual way: usual for the particular stands may convey title to such property, but the partnership or usual for similar partnerships. partnership may recover such property if the Actually, the acts mentioned in No. 1 refer only partners act does not bind the partnership under to acts of administration. the provisions of the first paragraph of Article 2.) Acts of strict dominion (pars. 2 and 3) For 1818, unless the purchaser or his assignee, is a acts which are not apparently for carrying on in holder for value, without knowledge. the usual way of business of the partnership, the Where the title to real property is in the name partnership is not bound, unless authorized by of one or more or all the partners, or in a third all the other partners or unless they have person in trust for the partnership, a conveyance abandoned the business. The general rule is that executed by a partner in the partnership name, or powers not specifically delegated in a in his name, passes the equitable interest of the partnership agreement are presumed to be partnership, provided the act is one within the withheld. Paragraph 3 gives instances of acts authority of the partner under the provisions of generally outside the implied power of a partner the first paragraph of Article 1818. and constitute limitations to the authority to Where the title to real property is in the names bind partnership. of all the partners a conveyance executed by all 3.) Acts in contravention of a restriction on the partners passes all their rights in such authority (par. 4) The partnership is not property. liable to third persons having actual or presumptive knowledge of the restrictions, DE-EMPHASIZE! whether or not the acts are for apparently carrying on in the usual way the business of the Title Deed Signatory Effect Exception partnership. ABC & ABC & A or B or Passes If act does On the other hand, persons not having such Co. Co. C legal title not bind notice have a right to assume that the authority partnership of a partner is co-extensive with the business under Art. transacted by the firm. 1818. ABC & ABC & A, B, and Passes No Liability of partner acting without authority Co. Co. C legal title exception As a general rule, the particular partner who undertakes to bind his co-partners by a contract without ABC & A or B A or B or Passes W/o authority is himself personally liable on such contract. Co. or C C equitable authority Such partner binds himself no matter in what name interest knowledge he contracts. The fact that he attempts to bind his co-A,B or Passes W/o partners and does not succeed does not avoid his own A,C or title authority act. He cannot be admitted to say that he was not B,C knowledge authorized to make a contract, as he is estopped to 1 or ABC & Passes W/o deny its effect or validity. more or Co. or equitable authority all or in his interest under art. Art. 1819. Where title to real property is in the trustee name 1818 partnership name, any partner may convey title to A, B A, B, Passes such property by a conveyance executed in the and C and C legal title partnership name; but the partnership may If this doesnt make sense, ok lang. de-emphasize recover such property unless the partners act naman eh! binds the partnership under the provisions of the first paragraph of article 1818, or unless such Effects of conveyance of real property belonging to property has been conveyed by the grantee or a the partnership person claiming through such grantee to a holder The ownership of real estate is prima facie that for value without the knowledge that the partner, indicated by the muniment of title. Ordinarily, title to in making the conveyance, has exceeded his real property or interest therein belonging to the authority. partnership is registered in the partnership name. Where title to real property is in the name of However, for one reason or another, the title to the the partnership, a conveyance executed by a property is not held by the partnership, although as partner, in his own name, passes the equitable between the partners there is no question that it is a

interest of the partnership, provided the act is one partnership property. The presumption is that, property within the authority of the partner under the purchased with partnership funds belongs to the provisions of the first paragraph of article 1818. partnership unless a contrary intent is shown. Where title to real property is in the name of Article 1819 gives the legal effects of the conveyance one or more but not all the partners, and the of real property belonging to the partnership depending in whose name it is registered and in whose name it is
Helen C. Arevalo 26 Section 3D

Finals Reviewer PARTNERSHIP 1st Sem; 2003


conveyed. Under the article, the real property may be EMPHASIZE! registered or owned in the name of: 1.) The partnership (pars. 1,2); Effects of admission by a partner 2.) One or more but not all the partners (par. 3); As a general rule, a person is not bound by the act, 3.) One or more or all the partners, or in a 3rd admission, statement, or agreement of another of which person in trust for the partnership (par. 4); he has no knowledge or to which he has not given his 4.) All the partners (par. 5). consent except by virtue of a particular relation between It will be noticed that under paragraphs 1, 3 and 5, them. what is conveyed is title or ownership, while under But admissions by a party as testified to by a 3rd paragraphs 2 and 4, what is conveyed is merely person are admissible in evidence against him in equitable interest. Conveyance interpreted to include a litigation. Admissions by another are received against a mortgage. Thus, the right to mortgage is included in the party if the former is acting in the capacity of agent of right to convey (unlike in agency). the latter. Thus, under Article 1820, the admission of a partner made during the existence of the partnership Innocent purchasers without notice are binding against the partnership (and co-partners) Regardless of the fact that one partner cannot when such admissions refer to a matter concerning convey partnership realty without the concurrence of his partnership affairs and made within the scope of his co-partners, it is fundamental that innocent purchasers authority. But when a partner makes no admissions for without notice may be protected. himself only without purporting to act for the Where the legal title is in the partner making the partnership, he alone shall be chargeable with his conveyance, although the equitable title is in the firm, a admissions. purchaser without notice may acquire a valid title, since After dissolution, admission made by a partner will he has the right to presume that possession or interest bind the co-partners if connected with the winding up of of the partnership is subordinate to and not inconsistent partnership affairs. with the record title. A conveyance by a partner of partnership property in Existence of partnership must be proved the partnership name even without authority, cannot be Before the partnership can be charged with the recovered by the partnership where it has been admissions of a partner under Article 1820, the conveyed by the grantee to a holder for value and partnership relation must be shown and proof of that without notice or knowledge that the partner, in making fact must be made by evidence other than the admission the conveyance, had exceeded his authority. itself. The purchaser need not have either actual or However, admissions and declarations made in the constructive notice of any trust or other condition presence of the person to be charged are admissible to limiting the authority of the partner making the prove the existence of the partnership. conveyance. An admission made by a partner who was no longer a partner at the time of the declaration is not admissible Authorization or ratification of conveyance in evidence against the partnership. A conveyance of partnership realty by one partner may be authorized by his co-partners, or when made Art. 1821. Notice to any partner of any matter without authority, may be ratified by them. Such relating to partnership affairs, and the knowledge authority or ratification must affirmatively appear, for of the partner acting in the particular matter, the authority of one partner to make and acknowledge a acquired while a partner or then present to his deed for the partnership will not be presumed. mind, and the knowledge of any other partner who After the lapse of many years, authority or reasonably could and should have communicated ratification will be presumed. it to the acting partner, operate as notice to or Authority may also be implied from the nature of the knowledge of the partnership except in case of a

partnership business. Where a firm is engaged in the fraud on the partnership, committed by or with the business of buying and selling real estate, the contract is consent of that partner. valid. Other partners will also be bound if there is DE-EMPHASIZE! subsequent adoption of the act. Ratification may be inferred from the presence of the Notice to, or knowledge of, a partner of matter other partners at the execution and delivery, or from affecting partnership affairs their acting under it or knowingly taking the benefits Notice to, or knowledge of, any partner of any matter arising therefrom. relating to partnership affairs operates as a notice to or knowledge of the partnership except in case of fraud. Art. 1820. An admission or representation made A 3rd person desiring to give notice to a partnership by any partner concerning partnership affairs of some matter pertaining to the partnership business within the scope of his authority in accordance need not communicate with all the partners. If notice is with this Title is evidence against the partnership. delivered to a partner, that is an effective communication to the partnership notwithstanding the
Helen C. Arevalo 27 Section 3D

Finals Reviewer PARTNERSHIP 1st Sem; 2003


failure of the partner to communicate such notice or Reason for imposition of wider liability Public knowledge to his co-partners. policy. The rule of respondeat superior (vicarious liability) applies. Cases of knowledge of a partner The obligation is solidary because the law protects 1.) Knowledge of the partner acting in the particular him who, in good faith, relied upon the authority of a matter acquired while a partner; partner, whether such authority is real or apparent. 2.) Knowledge of the partner acting in the particular Injured party may proceed against partnership matter then present to his mind; and or any partner Since the partners are liable 3.) Knowledge of any other partner who reasonably solidarily, the party aggrieved has his election to sue the could and should have communicated it to the firm or to sue one or more of its members. He may even acting partner. single out for suit a partner who, personally, was in no wise involved in the commission of the tort or breach of Art. 1822. Where, by any wrongful act or trust. omission of any partner acting in the ordinary Requisites for liability: course of the business of the partnership or with 1.) The partner must be guilty of a wrongful act or the authority of his co-partners, loss or injury is omission; and caused to any person, not being a partner in the 2.) He must be acting in the ordinary course of partnership, or any penalty is incurred, the business, or with the authority of his co-partners partnership is liable therefore to the same extent even if the act is unconnected with the business. as the partner so acting or omitting to act. The partnership is not liable if the partner acted on his own and not for the benefit of the partnership in the Art. 1823. The partnership is bound to make course of some transaction not connected with the good the loss: partnership business. 1.) Where one partner acting within the scope A non-acting partner in a partnership engaged in a of his apparent authority receives money or lawful business is not criminally liable for the criminal property of a third person and misapplied acts of another partner but he is criminally liable if the it; and partnership is involved in an unlawful enterprise with his 2.) Where the partnership in the course of its knowledge or consent. business receives money or property of a Criminal liability of partnership Partnership third person and the money or property so liability under Article 1822 does not extend to criminal received is misapplied by any partner while liability where the wrongdoing is regarded as individual it is in the custody of the partnership. in character. But where the crime is statutory, especially where it Art. 1824. All partners are liable solidarily with involves a fine rather than imprisonment, even criminal the partnership for everything chargeable to the liability may be imposed. partnership under Articles 1822 and 1823. Misapplication of money or property of a third 1822-4: EMPHASIZE! person Under article 1823, the partnership is liable for any Liability arising from partners tort or breach of losses suffered by a 3rd person whose money or property trust is misappropriated by a partner who received it within Nature of liability The above 3 articles provide the scope of his authority or by any other partner after it for the solidary liability of the partners and the was received by the partnership in the ordinary course partnership to 3rd persons for the wrongful act or of business while in its custody. omission or breach of trust of a partner acting within the scope of the firms business or with the authority of his Art. 1825. When a person, by words spoken or co-partners. This is true even though the other partners written or by conduct, represents himself, or did not participate in, or ratify, or had no knowledge of consents to another representing him to anyone, the act or omission. as a partner in an existing partnership or with one This liability of the partners is different from their or more persons not actual partners, he is liable to liability for contractual obligations as defined in Article any such persons to whom such representation 1816. Here, it is solidary, while in Article 1816, it is joint has been made, who has, on the faith of such and subsidiary. Furthermore, while the liability in Article representation given credit to the actual or 1816 refers to partnership obligations, this article covers apparent partnership, and if he has made such

the liability of the partnership arising from the wrongful representation or consented to its being made in a acts or omissions of any partner. The act or omission is public manner he is liable to such person, whether called tort when it does not constitute a crime or felony the representation has or has not been made or punishable by law. communicated to such person so giving credit by In workmens compensation cases, the liability of or with the knowledge of the apparent partner business partners arising from compensable injury or making the representation or consenting to its death of an employee should be solidary. being made:
Helen C. Arevalo 28 Section 3D

Finals Reviewer PARTNERSHIP 1st Sem; 2003


1.) When a partnership liability results, he is When liability pro rata When there is no existing liable as though he were an actual member partnership and all those represented as partners of the partnership; consented to the representation, or not all of the 2.) When no partnership liability results, he is partners of an existing partnership consented to the liable pro rata with the other persons, if representation, then, the liability of the person who any, so consenting to the contract or represented himself to be a partner or who consented to representation as to incur liability, his being represented as partner, and all those who otherwise separately. made and consented to such representation, is joint or When a person has been thus represented to be pro rata. a partner in an existing partnership, or with one or When liability separate When there is no more persons not actual partners, he is an agent existing partnership and not all but only some of those of the persons consenting to such representation represented as partners consented to the to bind them to the same extent and in the same representation, or none of the partners in an existing manner as though he were a partner in fact, with partnership consented to such representation, then the respect to persons who rely upon the liability will be separate that of the person who representation. When all the members of the represented himself as a partner or who consented to existing partnership consent to the representation, his being represented as a partner, and those who made a partnership act or obligation results; but in all and consented to the representation, or that only of the other cases it is the joint act or obligation of the person who represented himself as partner. person acting and the persons consenting to the Estoppel does not create partnership Only representation. liability is created. Liability as partners may arise contrary to their EMPHASIZE! COMMENTARY (not so much codal) intentions The question of liability is not what the Important: Difference between partner by estoppel and parties intended by their contract but whether 3rd partnership by estoppel. persons had a right to rely on their joint credit. One who is deemed to be liable as a partner by Partner by estoppel; partnership by estoppel reason of estoppel does not thereby obtain full rights as Meaning and effect of estoppel Estoppel is a bar a partner. which precludes a person from denying or asserting Application of estoppel as between partners anything contrary to that which has been established as The doctrine of estoppel has no application as between the truth by his own deed or representation, either actual partners. Partners become such by agreement express or implied. Through estoppel, an admission or and not by estoppel. It is true that a single partner may representation is rendered conclusive upon the person become liable to 3rd persons beyond the limits fixed by making it and cannot be denied or disapproved as the partnership agreement by holding out as partners to against the person relying thereon. an extent greater than that specified in the partnership When person a partner by estoppel A person agreement. As between the partners, such an action not a partner may become a partner by estoppel and might be the basis for a dissolution of the partnership thus be held liable to 3rd persons as if he were a partner, but it would apply primarily to 3rd persons who had when by words or by conduct he: acted on the representations to their detriment. 1.) Directly represents himself to anyone as a Application of estoppel as to third parties It is partner in an existing partnership or in a non-in this area that the doctrine of estoppel has been existing partnership (with one or more persons applied. not actual partners). By the person himself; or Applicability of general provisions on 2.) Indirectly represents himself by consenting to partnership If the law recognizes a defectively another representing him as a partner in an organized partnership as de facto as far as 3rd persons existing partnership or in an non-existing are concerned, for purposes of its de facto existence, it partnership. By his consent or with his should have such attribute of a partnership as domicile. knowledge. Although it has no legal standing or juridical personality, To hold the party liable, the 3rd person must prove: it is a partnership de facto and the general provisions of

1.) The misrepresentation; and the Civil Code applicable to partnerships apply to it. 2.) That a bona fide reliance by him upon it caused him injury. Elements to establish liability as a partner on When partnership liability results If all the ground of estoppel actual partners consented to the representation, then 1.) Proof by plaintiff that he was individually aware the liability of the person who represented himself to be of the defendants represe ntations as to his a partner or who consented to such representation and being a partner or that such representations the actual partners is considered a partnership liability. were made by others and not denied or refuted This is a case of partnership by estoppel. The person by the defendant; becomes an agent of the partnership and his act or 2.) Reliance on such representations by the plaintiff; obligation that of the partnership. and
Helen C. Arevalo 29 Section 3D

Finals Reviewer PARTNERSHIP 1st Sem; 2003


3.) Lack of any denial or refutation of the Reason for rule making the new partner liable statements by the defendant. New partner partakes also of the benefits of the partnership property and an established business. He Liability as general partners of persons who has every means of obtaining full knowledge of the assume to act as a corporation debts of the partnership and protecting himself because The law makes liable as general partners all persons he may insist on the liquidation or settlement of existing who assume to act as a corporation and may include partnership debts. On the other hand, these means are persons who attempt but fail to form a corporation and not afforded the creditor. who carry on business under the corporate name. A de facto partnership among them is created. Only the Liability of outgoing partner / incoming partner active members of the unsuccessfully attempted Contract made before retirement or withdrawal corporation should be liable as general partners. Where a partner gives notice of his retirement or Subscribers to stocks who take no part in the supposed withdrawal from the partnership, he is freed from any corporation are not personally liable. liability on contracts entered into thereafter, but his liability on existing incomplete contracts continues. Art. 1826. A person admitted as a partner into Thus, he is liable for goods sold and delivered after his an existing partnership is liable for all the retirement or withdrawal and notice thereof, if the same obligations of the partnership arising before his was pursuant to a contract made before such retirement admission as though he had been a partner when or withdrawal. such obligations were incurred, except that this Performance after admission of new partner liability shall be satisfied only out of partnership In the case of an incoming partner, he is not personally property, unless there is a stipulation to the liable for the existing partnership obligations unless contrary. there is a stipulation to the contrary. But he is liable for goods delivered to the partnership after his admission to Liability of incoming partner for partnership it, where the goods so delivered are in the performance obligations of a contract made before his admission. The result is Limited to his share in partnership property for that both the retiring and the incoming persons are existing obligations When a partner is admitted as a liable for the debt created by delivery of such goods. partner into an existing partnership, he is liable for all obligations existing at the time of his admission as Art. 1827. The creditors of the partnership shall though he was already a partner when such obligations be preferred to those of each partner as regards were incurred. For such obligations, his liability is limited the partnership property. Without prejudice to this to his share in the partnership property, unless there is right, the private creditors of each partner may a stipulation to the contrary. ask the attachment and public sale of the share of Extends to his separate property for subsequent the latter in the partnership assets. obligations Those who were already partners at the time when the obligations were incurred are liable with Preference of partnership creditors in partnership their separate property. For all the obligations accruing property subsequent to the admission of the new partner, all the With respect to partnership assets, the partnership partners are liable with their separate properties. creditors are entitled to priority of payment. The rule applies only in the event of the disposition of Rights of existing and subsequent creditors partnership property among its creditors to pay Existing and subsequent creditors have equal rights partnership debts. The partners may deal with as against partnership property and separate property of partnership property in the usual course of business as the previously existing members of the partnership while they see fit. only subsequent creditors have rights against the separate estate of the newly admitted partner. Remedy of private creditors of a partner Where business is continued Where there is one Without prejudice to the right of preference of continuous business, all the creditors of the partnership creditors, the, the creditors of each partner partnership, irrespective of the times when they became may ask for the attachment and public sale of the share

creditors and the exact combinations of persons owning of the latter in the partnership assets. Such share really the business should have equal rights in property belongs to the partner. employed in the business. The purchaser at the public sale does not become a Where incoming partner has assumed partner. obligation of retiring partner If an incoming partner has assumed the obligation of the retiring partner as one of the terms of the contract by which he was admitted into the firm, he is liable directly to the old CHAPTER 3. DISSOLUTION AND WINDING UP partnership creditors if the assumption was made primarily to benefit the firm creditors (stipulation pour Sources of provisions: Uniform Partnership Act autrui).
Helen C. Arevalo 30 Section 3D

Finals Reviewer PARTNERSHIP 1st Sem; 2003


Art. 1828. The dissolution of a partnership is the property, means and assets of the the change in the relation of the partners caused partnership existing at the time of its dissolution by any partner ceasing to be associated in the for the benefit of all interested. carrying on as distinguished from the winding up 3.) Transaction of new business prohibited of the business. Upon dissolution, no new partnership business should be undertaken, but affairs should be EMPHASIZE: EFFECTS. liquidated and distribution made to those entitled to the partners interest. Effects of change in membership of a partnership It is only after winding up is accomplished that the 1.) Dissolution of existing partnership and existence of the partnership is terminated. Thus, formation of a new one Any change in the dissolution refers to the change in partnership relation membership of a partnership produces technically and not the actual cessation of the partnership business. an immediate dissolution of the existing It is not necessarily followed by a winding up of partnership relation, and the formation of a new partnership affairs. one, although common business usage speaks of Dissolution of a partnership must be distinguished the admission of a partner to a firm and regards from a mere suspension in the conduct of its business or the firm as subsisting so long as the course of its operation. business is not materially interrupted. 2.) Transformation of all partners into incoming Art. 1830. Dissolution is caused: partners All persons forming the new 1.) Without violation of the agreement partnership upon admission of a new person are between the partners: technically incoming partners even though the a.) By the termination of the definite same business had theretofore been conducted term or particular undertaking by the others through the medium of partnership. specified in the agreement; 3.) Continuance by remaining partners of b.) By the express will of any partner, partnership as before The change in the who must act in good faith, when no relation of the partners will dissolve the definite term or particular partnership but will not disturb the continuance undertaking is specified; by the remaining partners or by the existing and c.) By the express will of all the new partners of the business as before. partners who have not assigned their interests or suffered them to Dissolution, winding up, and termination defined be charged for their separate debts, Dissolution: The change in the relation of the either before or after the partners caused by any partner ceasing to be associated termination of any specified term or in the carrying on of the business. It is that point in time particular undertaking; when the partners cease to carry on the business d.) By the expulsion of any partner from together. It represents the demise of a partnership. the business bona fide in accordance Winding up: The process of settling the business or with such a power conferred by the partnership affairs after dissolution. agreement between the partners; Termination: That point in time when all partnership 2.) In contravention of the agreement between affairs are completely wound up and finally settled. It the partners, where the circumstances do signifies the end of the partnership life. not permit a dissolution under any other provision of this article, by the express will Art. 1829. On dissolution the partnership is not of any partner at any time; terminated, but continues until the winding up of 3.) By any event which makes it unlawful for partnership affairs is completed. the business of the partnership to be carried out on or for the members to carry EMPHASIZE! it on in partnership; 4.) When a specific thing, a partner had Effects of dissolution promised to contribute to the partnership, 1.) Partnership not terminated Dissolution does perishes before the delivery; in any case by not automatically result in the termination of the the loss of the thing, when the partner who legal personality of the partnership, nor the contributed it having reserved the relations of the partners among themselves who ownership thereof, has only transferred to remain as co-partners until the partnership is the partnership the use or enjoyment of the

terminated. same; but the partnership shall not be 2.) Partnership continues for a limited purpose dissolved by the loss of the thing when it After dissolution, a partnership is considered as occurs after the partnership has acquired maintaining a limited existence for the purpose of the ownership thereof; making good all outstanding engagements, of 5.) By the death of any partner; taking and settling all accounts, and collecting all
Helen C. Arevalo 31 Section 3D

Finals Reviewer PARTNERSHIP 1st Sem; 2003


6.) By the insolvency of any partner or of the dissolution may be accomplished either by an partnership; express agreement or by words and acts implying 7.) By the civil interdiction of any partner; an intention to dissolve. The agreement to 8.) By decree of court under the following dissolve before the termination of term/particular article. undertaking must be unanimous. But those who have assigned their interests or have suffered EMPHASIZE! them to be charged for their separate debts, do not get to vote. Causes of dissolution 4.) By expulsion of any partner The expulsion Statutory enumeration exclusive Articles 1830 must be made in good faith, and strictly in and 1831 provide for the causes of dissolution. Other accordance with the power conferred by the causes are provided in Article 1840. agreement between the partners. The partner Under Article 1830, extrajudicial dissolution may be expelled in bad faith can claim damages. caused without violation of the agreement between the partners (No. 1) or in contravention of said agreement Dissolution effected in contravention of (No. 2). It may be voluntary when caused by the will of partnership agreement one or more or all of the partners (Nos. 1 and 2) or Dissolution may be for any cause or reason involuntary when brought about independently of the Any partner may cause the dissolution of the partnership will of the partners or by operation of law (Nos. 3, 4, 5, at any time w/o consent of his co-partners for any 6, 7 and 8). reason which he deems sufficient by expressly The voluntary dissolution of partnership may be withdrawing therefrom even though the partnership was effected extrajudicially (Nos. 1 to 7) or judicially, that is, entered into for a definite term or particular by decree of court. (No. 8, in relation to Art. 1831.) It undertaking. Dissolution of such partnership is, will be observed that the causes provided in Article 1830 however, a contravention of the agreement. result in the automatic dissolution of the partnership. The legal effects of this dissolution are liad down in The statutory enumeration of the causes of article 1837, par. 2, Nos. 1, 2 and 3. dissolution precludes dissolution for any other cause. Power of dissolution always exists There is no Effect of sale or assignment by one partner of such thing as an indissoluble partnership in the sense his entire interest in the partnership to a third that there is always the power of dissolution. The person It does not ipso facto bring about the doctrine of delectus personae allows the partners to dissolution of the partnership. That it produces have the power, although not necessarily the right, to dissolution may be inferred, however, from the definition dissolve the partnership. An unjustified dissolution by a of dissolution under Article 1828. But the dissolution partner can subject him to a possible action for created in such case is only technical, and not actual, damages. i.e., only in the sense that his connection with the partnership is terminated. Business becomes unlawful In practice, the SEC accepts for registration amended Dissolution may be caused involuntarily when a articles of partnership together with the deed of sale of supervening event makes the business itself of the the interest of the withdrawing partner. partnership unlawful or makes it unlawful for the partners to carry it on together. A partnership must Dissolution effected without violation of have a lawful object or purpose. partnership agreement There are 4 ways by which a partnership may be Loss of specific thing dissolved without violation of the partnership The provision refers only to specific things. When the agreement: thing to be contributed is not specific, Articles 1786 1.) Termination of the definite term or (par. 1) and 1788 shall govern. particular undertaking After the expiration of Loss before delivery Partnership is dissolved the term of particular undertaking, the because there is no contribution inasmuch as the thing partnership is automatically dissolved. If after to be contributed cannot be substituted with another. said expiration, they continue w/o making a new There is here a failure of a partner to fulfill his part of agreement, a partnership at will is created. the obligation. 2.) By the express will of any partner A Loss after delivery Partnership not dissolved but

partnership at will may be dissolved at any time it assumes the loss of the thing having acquired by any partner w/o the consent of his co-partners ownership thereof. The partners may contribute w/o breach of contract, provided, the said additional capital to save the venture. partner acts in good faith. If there is bad faith, Loss where only the use or enjoyment the dissolution is wrongful and the dissolving contributed Loss before or after delivery dissolves partner will be liable for damages. the partnership because in either case, the partner 3.) By the express will of all the partners No cannot fulfill his undertaking to make available the use particular form of agreement is necessary to of the specific thing contributed. Here, the contributing dissolve a partnership by consent. Such partner bears the loss since he retains ownership and,
Helen C. Arevalo 32 Section 3D

Finals Reviewer PARTNERSHIP 1st Sem; 2003


therefore, he is considered in default with respect to his conveyance inter vivos. One who is w/o capacity to contribution. Upon dissolution, the partners may manage his own property should not be allowed to demand for an accounting and liquidation. manage partnership property. The mere failure by a partner to contribute his share of capital does not prevent the existence of a firm. Such Right to expel a partner failure may be waived by the others. In the absence of an express agreement to that effect, there exists no right or power of any member, or Death of any partner even a majority of the members, to expel all other The deceased partner ceases to be associated in the members of the firm at will. Nor can they at will forfeit carrying on of the business; hence, the ipso facto the share or interest of a member and compel him to dissolution of the partnership by his death by operation quit the firm, even paying what is due him. of law. The surviving partners have no authority to Partner guilty of extreme and gross faults continue the business except as provided in Article Mere derelictions do not ipso facto forfeit his right to the 1833. common property or assets of the partnership. There Status of partnership Subsequent legal status of may be, however, extreme and gross faults which would partnership is that of a partnership in liquidation, and work a forfeiture, especially where there was an the only rights inherited by the heirs are those resulting extreme emergency for him to perform his duty, and to from said liquidation. Before liquidation is made, it is be prompt and faithful. impossible to determine the share of the deceased Industrial partners, engaging in business for partner. himself The law authorizes the capitalist partners to Liquidation of its affairs The liquidation is exclude an industrial partner who engages in business entrusted to the surviving partners, or to liquidators for himself w/o express permission of the partnership. appointed by them and not the administrator or Power expressly given by agreement A power executor of the deceased partner. of expulsion may be expressly given by agreement. But Continuation of business without liquidation A the power is not validly exercised if it is shown to have clause in the articles of partnership providing for the been exercised unfairly and w/o regard to the general continuance of the firm notwithstanding the death of one interest of the partnership. of the partners is legal. By common agreement, the surviving partners and Art. 1831. On application by or for a partner, the heirs of the deceased may decide to continue the the court shall decree a dissolution whenever: partnership. But they become liable to the old creditors 1.) A partner has been declared insane in any of the firm. judicial proceeding or is shown to be of unsound mind; Insolvency of any partner or of partnership 2.) A partner becomes in any other way The insolvency of the partner or of the partnership incapable of performing his part of the must be adjudged by a court. partnership contract; The insolvency of a partner subjects his interest in 3.) A partner has been guilty of such conduct the partnership to the right of his creditors and makes it as tends to affect prejudicially the carrying impossible for him to satisfy with his property on of the business; partnership obligations to its creditors in the event that 4.) A partner willfully or persistently commits a partnership assets have been exhausted. Thus, by his breach of the partnership agreement, or insolvency, its credit is impaired. An insolvent partner otherwise so conducts himself in matters has no authority to act for the partnership not the other relating to the partnership business that it partners to act for him. is not reasonably practicable to carry on The insolvency of the partnership renders its property the business in partnership with him; in the hands of the partners liable for the satisfaction of 5.) The business of the partnership can only be partnership obligations resulting in their inability to carried on at a loss; continue the business, which practically amounts to a 6.) Other circumstances render a dissolution dissolution. But the reconveyance by the assignee of the equitable; properties of the partnership pursuant to an order of the On the application of the purchaser of a court after the termination of the insolvency proceedings partners interest under Article 1813 or 1814:

involving the partnership has the effect of restoring the 1.) After the termination of the specified partnership to its status quo. term or particular undertaking; 2.) At any time if the partnership was a Civil interdiction of any partner partnership at will when the interest was A partnership requires the capacity of the partners. A assigned or when the charging order was person under civil interdiction cannot validly give issued. consent, as his capacity to act is limited thereby. Civil interdiction deprives the offender during the EMPHASIZE! time of his sentence of the right to manage his property and dispose of such property by any act or any Judicial determination as to dissolution
Helen C. Arevalo 33 Section 3D

Finals Reviewer PARTNERSHIP 1st Sem; 2003


Events which make it impossible to carry on the Art. 1832. Except so far as may be necessary to business as intended may have such serious effect that wind up partnership affairs or to complete the partnership ought to be dissolved by decree of the transactions begun but not then finished, court. Such events as unlawfulness, death, or insolvency dissolution terminates all authority of any partner of a partner are certain and unequivocal. Their to act for the partnership: occurrence and effect is not a matter of dispute or 1.) With respect to the partners, doubt. a.) When the dissolution is not by the On the other hand, the facts may be so far open to act, insolvency or death of a dispute as to make a necessary judicial determination as partner; or to dissolution rather than allow them to be the occasion b.) When the dissolution is by such act, for automatic dissolution by operation of law. Among the insolvency or death or a partner, in facts or acts which will warrant a dissolution by judicial cases where Article 1833 so decree are those enumerated in the 1st paragraph of requires; article 1831. 2.) With respect to persons not partners, as declared in Article 1834. Grounds for dissolution by decree of court Dissolution of a partnership may be decreed judicially Effect of dissolution on authority of partner on application, either (1) by a partner in the cases General rule Upon dissolution, the partnership mentioned in paragraph 1, Nos. 1-6; or (2) by the ceases to be an on-going concern and the partners purchaser or assignee of a partners interest under power of representation is confined only to acts incident paragraph 2, Nos. 1 and 2. to winding up or completing transactions begun but not On application by a partner: then finished. Insanity The partner may have been previously The event of dissolution terminates the actual declared insane in a judicial proceeding; otherwise, the authority of a partner to undertake new business for the fact of his being of unsound mind must be duly proved. partnership. Incapacity This refers to incapacity other than Qualifications to the rule: insanity. The incapacity must be lasting from which the Insofar as the partners themselves are prospect of recovery is remote. If the disability is merely concerned, the authority of any partner to bind the temporary, there is no fit ground to decree a dissolution. partnership by a new contract is immediately terminated Misconduct and persistent breach of when the dissolution is not by the act, insolvency, or partnership agreement like incapacity, conduct death of a partner. When the dissolution is by such act, prejudicial to the carrying on of the business and insolvency, or death, the termination of authority persistent breach of the partnership agreement are depends upon whether or not the partner had grounds for judicial dissolution, for they defeat and knowledge or notice of the dissolution as provided in materially affect and obstruct the purpose of the Article 1833. partnership. With respect to third persons, the partnership Again, temporary grievances, etc., will not suffice. is generally bound by the new contract although the But courts can order dissolution over petty arguments authority of the acting partner as it effects his cowhen these are continuous and to such an extent that all partners is already deemed terminated. However, the confidence and cooperation is gone. innocent partners may recover from the acting partner. Business can be carried on only at a loss Since the purpose of the partnership is the carrying of a Art. 1833. Where the dissolution is caused by business for profit, it may be dissolved by decree of the act, death or insolvency of partner, each court when it becomes apparent that it is unprofitable partner is liable to his co-partners for his share of with no reasonable prospects of success. any liability created by any partner acting for the A court is authorized to decree a dissolution partnership as if the partnership had not been notwithstanding that the partnership has been making dissolved unless: profit where it appears at the time of the application 1.) The dissolution being the act of any that the business can only be carried on at a loss. partner, the partner acting for the

Other circumstances Examples of partnership had knowledge of the circumstances which render a dissolution equitable are dissolution; or abandonment of the business, fraud in the management 2.) The dissolution being by the death or of the business, refusal w/o justifiable cause to render insolvency of a partner, the partner acting accounting of partnership affairs, etc. for the partnership had knowledge or On application by a purchaser of a partners notice of the death or insolvency. interest In either of the two cases mentioned in the last paragraph, a purchaser of a partners interest under NEVER MIND THIS! Article 1813 or 1814 may apply for judicial dissolution of a partnership. Right of partner to contribution from co-partners The above article speaks of dissolution caused by act, insolvency, or death of a partner.
Helen C. Arevalo 34 Section 3D

Finals Reviewer PARTNERSHIP 1st Sem; 2003


Where a partner enters into a new contract with a 3rd 1.) Unknown as a partner to the person with person after dissolution, partners generally bound. whom the contract is made; and 2.) So far unknown and inactive in Authority of partners inter se to act for the partnership affairs that the business partnership reputation of the partnership could not be The authority of a partner as it affects his co-partners said to have been in any degree due to his (not 3rd persons) is deemed terminated except in Nos. 1 connection with it. and 2 of 1833. The partnership is in no case bound by any act of a partner after dissolution: Knowledge or notice of cause of dissolution 1.) Where the partnership is dissolved Dissolution by death or insolvency When because it is unlawful to carry on the partner dead or bankrupt and other partner did not business, unless the act is appropriate for know it when he entered into transaction, he may call on winding up partnership affairs; or other partners to contribute. 2.) Where the partner has become insolvent; Dissolution by court decree or resulting from or unlawfulness No problem exists in these cases. If by 3.) Where the partner has no authority to court decree, all partners have actual notice of wind up partnership affairs, except by a dissolution. If due to unlawfulness, general rules transaction with one who governing actions arising out of illegal transactions a.) Had extended credit to the apply. partnership prior to dissolution and had no knowledge or notice of his When a partner has knowledge or notice of a fact want of authority; or Uniform Partnership Act: b.) Had not extended credit to the Knowledge: Not only actual but also knowledge of partnership prior to dissolution, and, such other facts as in the circumstances show bad faith. having no knowledge or notice of his Notice: When the person who claims the benefit of want of authority, the fact of his the notice: want of authority has not been 1.) States the fact to such person; or advertised in the manner provided 2.) Delivers through the mail or by other means of for advertising the fact of dissolution communication, a written statement of the fact in the first paragraph, No. 2. or to a proper person at his place of business or Nothing in this article shall effect the liability residence. under Article 1825 of any person who after dissolution represents himself or consents to Art. 1834. After dissolution, a partner can bind another representing him as a partner in a the partnership except as provided in the third partnership engaged in carrying in business. paragraph of this article: 1.) By any act appropriate for winding up NOT NECESSARILY CODAL BUT COMMENTARY IMPT. partnership affairs or completing transactions unfinished at dissolution; or Power of partners to bind dissolved partnership to 2.) By any transaction which would bind the third persons partnership if dissolution had not taken Article 1834 enumerates the cases when a partner place, provided the other party to the continues to bind the partnership even after dissolution transaction: (par. 1, Nos. 1 and 2) and the case when he cannot bind a.) Had extended credit to the the partnership after dissolution (par. 3, Nos. 1, 2 and partnership prior to dissolution and 3). had no knowledge or notice of the Where there is no notice to third persons of dissolution; or dissolution Upon dissolution of the partnership, as b.) Though he had not so extended between themselves, the power of one partner to act credit, had nevertheless known of and bind the others is effectively terminated. But the the partnership prior to dissolution, authority of a partner may apparently continue as and, having no knowledge or notice regards 3rd persons on the assumption that the of dissolution, the fact of dissolution partnership is still existing. Since a partnership once had not been advertised in a established is, in the absence of anything to indicate its newspaper of general circulation in termination, presumed to exist, the law, for the the place (or in each place if more protection of innocent 3rd persons, imposes upon than one) at which the partnership partners the duty of giving notice of the dissolution of business was regularly carried on. the partnership.

The liability of a partner under the first Where there is actual or constructive paragraph, No. 2, shall be satisfied out of knowledge by third persons of dissolution The partnership assets alone when such partner had measure of the right of 3rd persons who continue to deal been prior to dissolution: with a dissolved partnership depends upon the question
Helen C. Arevalo 35 Section 3D

Finals Reviewer PARTNERSHIP 1st Sem; 2003


of whether they knew or should have known of the fact Since dormant partner never known or held out to be of dissolution. If they did, the validity of their a partner, 3rd persons, not having dealt with the transactions is governed by the question whether those partnership in reliance upon the membership of the transactions were necessary to liquidate the partnership dormant partner, are not entitled to notice of his affairs. withdrawal. Notice of dissolution to creditors Partnership by estoppel after dissolution As to persons who extended credit to Article 1834 (last par.) touches upon the subject of partnership prior to dissolution Must have partnership be estoppel (Art. 1825), since a partnership knowledge or notice of the dissolution to relieve is held to exist as to 3rd persons though it does not exist partnership from liability. as a going concern so far as the partners themselves are As to persons who had not extended credit prior concerned. The situation differs from a partnership by to dissolution but had known of partnerships estoppel, however, in that a partnership did once exist existence Fact that dissolution had been published in and liability is based on its continuance as a matter of the newspaper sufficient even if they did not actually law as far as 3rd persons are concerned. A partnership read the advertisement. by estoppel involves a holding out by parties as partners Where acting partner has no authority to wind when, in fact, they are not partners. up partnership affairs under the 3rd paragraph, notice of dissolution is unnecessary except in case No. 3, Art. 1835. The dissolution of the partnership where the partner has no authority to wind up does not of itself discharge the existing liability of partnership affairs. 3rd persons dealing with the partner any partner. w/o such authority are protected under the same A partner is discharged from any existing circumstances mentioned in paragraph 1, No. 2 (a) and liability upon dissolution of the partnership by an (b). agreement to that effect between himself, the Where acting partner has become insolvent partnership creditor and the person or partnership Innocent partner (did not know of other partners continuing the business; and such agreement may insolvency) is protected in his continued right to make be inferred from the course of dealing between the binding partnership agreements, but no similar creditor having knowledge of the dissolution and protection is extended to a 3rd person who innocently the person or partnership continuing the business. contracts with an insolvent partner because it is The individual property of a deceased incumbent upon him to know the status of the insolvent partner shall be liable for all obligations of the partner. partnership incurred while he was a partner, but Where dissolution caused by death of a partner subject to the prior payment of his separate debts. Death is not considered to be notice per se whether as to surviving partner or as to 3rd persons. Effect of dissolution on partners existing liability The dissolution of a partnership does not of itself Character of notice required discharge the existing liability of a partner. The character of notice required to relieve a retiring A partner may be relieved from all existing liabilities partner or the representatives of a deceased partner upon dissolution only by an agreement to that effect from subsequent liability on partnership obligations between himself, the partnership creditor, and the other varies in accordance with the class of persons required partners. The consent, however, of the creditor and the to be notified. other partners to the novation may be implied from their As to prior dealers Notice must be actual. mere conduct. mailing of a letter is insufficient if notice never received. Furthermore, there is no duty on the part of the prior Liability of estate of deceased partner dealer to inquire into the question of retirement. That In accordance with Article 1816, the individual the retirement was mentioned in a newspaper is property of a deceased partner shall be liable for all

insufficient. obligations of the partnership incurred while he was a A prior or former dealer is one who has extended partner. Note that the individual creditors of the credit on the faith of the partnership. Mere dealing with deceased partner are to be preferred over partnership the firm on a cash basis does not constitute one as a creditors with respect to the separate property of said prior dealer. deceased partner. As to all others Actual notification not necessary. Advertisement in local newspaper enough. It should be Art. 1836. Unless otherwise agreed, the noted, however, that the requirement of newspaper partners who have not wrongfully dissolved the notice appears to exist only where the 3rd party knew of partnership or the legal representative of the last the partnership prior to dissolution. If not, he is entitled surviving partner, not insolvent, has the right to to no notice whatsoever. wind up the partnership affairs, provided, however, that any partner, his legal representative Dormant partner need not give notice or his assignee, upon cause shown, may obtain winding up by the court.
Helen C. Arevalo 36 Section 3D

Finals Reviewer PARTNERSHIP 1st Sem; 2003


respect of their interests in the partnership, unless Manner of winding up otherwise agreed, may have the partnership The manner of winding up of the dissolved property applied to discharge its liabilities, and the partnership may be done either: surplus applied to pay in cash the net amount 1.) Judicially Under the control and direction of owing to the respective partners. But if dissolution the proper court upon cause shown by any is caused by expulsion of a partner, bona fide partner, his legal representative, or his assignee. under the partnership agreement and if the 2.) Extrajudicially By the partners themselves expelled partner is discharged from all partnership w/o intervention of the court. liabilities, either by payment or agreement under the second paragraph of Article 1835, he shall Nature of action for liquidation receive in cash only the net amount due him from An action for the liquidation of a partnership is a the partnership. personal one; hence, it may be brought in the place of When dissolution is caused in contravention of residence of either the plaintiff or the defendant. the partnership agreement the rights of the partners shall be as follows: Persons authorized to wind up 1.) Each partner who has not caused 1.) The partners designated by agreement; dissolution wrongfully shall have: 2.) In the absence of such agreement, all the a.) All the rights specified in the first partners who have not wrongfully dissolved the paragraph of this article, and partnership; or b.) The right, as against each partner 3.) The legal representative (executor or who has caused the dissolution administrator) of the last surviving partner (when wrongfully, to damages for breach of all the partners are already dead), not insolvent. the agreement. 4.) Court-appointed receiver. 2.) The partners who have not caused the dissolution wrongfully, if they all desire to Survivors right and duty to liquidate continue the business in the same name When a member of a partnership dies, the duty of either by themselves or jointly with liquidating its affairs devolves upon the surviving others, may do so, during the agreed term member(s) of the firm, not upon the legal representative for the partnership and for that purpose of the deceased partner. The latter has no right to may possess the partnership property, interfere so long as the surviving partner proceeds in provided they secure the payment by good faith. bond approved by the court, or pay to any partner who has caused the dissolution Powers of liquidating partner wrongfully, the value of his interest in the 1.) Make new contracts For the purpose of partnership at the dissolution, loss any winding up the partnership, a liquidating partner damages recoverable under the second is sole agent of the partnership, but merely for paragraph, No. 1 (b) of this article, and in that one specific purpose. He cannot make new like manner indemnify him against all contracts w/o express authority. present or future partnership liabilities. 2.) Raise money to pay partnership debts For 3.) A partner who has caused the dissolution the purpose of winding up the concern, however, wrongfully shall have: the liquidating partner may bind the partnership a.) If the business is not continued by borrowing money to meet its accruing under the provisions of the second liabilities, and may sell its real estate to raise paragraph, No. 2, all the rights of a money to pay its debts. partner under the first paragraph, 3.) Incur obligations to complete existing subject to liability for damages in the contracts or preserve partnership assets A second paragraph, No. 1(b), of this liquidating partner has power to incur obligations article. necessary to the completion of existing contracts, b.) If the business is continued under and to incur debts or other obligations necessary the second paragraph, No. 2, of this for the reasonable preservation of partnership article, the right as against his assets or in procuring a favorable market for copartners and all claiming through their disposal. them in respect of their interests in 4.) Incur expenses necessary in the conduct of the partnership, to have the value of litigation He has power to employ an attorney his interest in the partnership, less when necessary for winding up of affairs. any damage caused to his copartners by the dissolution,

Art. 1837. When dissolution is caused in any ascertained and paid to him in cash, way, except in contravention of the partnership or the payment secured by a bond agreement, each partner, as against his co-approved by the court, and to be partners and all persons claiming through them in released from all existing liabilities
Helen C. Arevalo 37 Section 3D

Finals Reviewer PARTNERSHIP 1st Sem; 2003


of the partnership; but in Firm name as part of goodwill The name of a ascertaining the value of the firm is an important part of the good will and its use partners interest the value of the may be protected accordingly. goodwill of the business shall not be Existence of a saleable goodwill The goodwill of considered. a business is a proper subject of sale. However, a saleable goodwill can exist only in a commercial DEINS DA DAW TO! partnership (not in a professional one). Right of partner to application of partnership Art. 1838. Where a partnership contract is property on dissolution rescinded on the ground of the fraud or The objectives of article 1837 are, in the main, to misrepresentation of one of the parties thereto, provide for the payment of the partner who leaves the the party entitled to rescind is, without prejudice firm, and to indemnify him against existing or possible to any other right, entitled: future liability. A partners lien is created. 1.) To a lien on, or right of retention of, the surplus of the partnership property after Rights where dissolution not in contravention of satisfying the partnership liabilities to 3rd agreement persons for any sum of money paid by him 1.) To have partnership property applied to for the purchase of an interest in the discharge the liabilities of the partnership; and partnership and for any capital or advances 2.) To have the surplus, if any, applied to pay in cash contributed by him; the net amount owing to the respective partners. 2.) To stand, after all liabilities to 3rd persons have been satisfied, in the place of the Rights where dissolution in contravention of creditors of the partnership for any agreement payments made by him in respect of the Rights of partner who has not caused the partnership liabilities; and dissolution wrongfully: 3.) To be indemnified by the person guilty of 1.) To have partnership property applied for the fraud or making the representation against payment of its liabilities and to receive in cash all debts and liabilities of the partnership. his share of the surplus; 2.) To be indemnified for damages caused by the DEINS NA DIN TO! partner guilty of wrongful dissolution; 3.) To continue the business in the same name Right of partner to rescind contract of partnership during the agreed term of the partnership, by If one is induced by fraud or misrepresentation to themselves or jointly with others; and become a partner, the contract is voidable or annullable. 4.) To possess partnership property should they If the contract is annulled, the injured party is decide to continue the business. entitled to restitution. Here, the fraud or Rights of partner who has wrongfully caused misrepresentation vitiates consent. However, until the the dissolution: partnership contract is annulled by a proper action in 1.) If business not continued by the other partners: court, the partnership relations exist and the defrauded to have the partnership property applied to partner is liable for all obligations to 3rd persons. discharge its liabilities and to receive in cash his share of the surplus less damages caused by his Rights of injured partner where partnership wrongful dissolution. contract rescinded 2.) If the business is continued: 1.) Right of lien on, or retention of, the surplus of a.) To have the value of his interest in the partnership property after satisfying partnership partnership at the time of the liabilities for any sum of money paid or dissolution, less any damage caused by contributed by him; the dissolution to his co-partners, 2.) Right to subrogation in place of partnership ascertained and paid in cash or secured creditors after payment of partnership liabilities; by bond approved by the court; and and b.) To be released from all existing and 3.) Right of indemnification by the guilty partner future liabilities of the partnership. against all debts and liabilities of the partnership. Goodwill of a business: The advantage which it has from its establishment or from the patronage of its Art. 1839. In settling accounts between the customers, over and above the mere value of its partners after dissolution, the following rules shall

property and capital. be observed, subject to any agreement to the Goodwill as part of partnership assets Good contrary: will of partnership, if of money value, is usually 1.) The assets of the partnership are: considered part of the property and assets of the firm, in a.) The partnership property, the absence of a contract to the contrary.
Helen C. Arevalo 38 Section 3D

Finals Reviewer PARTNERSHIP 1st Sem; 2003


b.) The contributions of the partners to one or more of their number as liquidating partner or necessary for the payment of all the partners. liabilities specified in No. 2. The law, however, does not require a partnership to 2.) The liabilities of the partnership shall rank convert all its assets into cash before making a in order of payment, as follows: distribution to the partners. It is within the power of the a.) Those owing to creditors other than court to order a distribution of its assets in cash, partners, property, or a combination of both. b.) Those owing to partners other than Property which may be made available for for capital and profits, distribution includes, in addition to the partnership c.) Those owing to partners in respect of property, contributions which may be collected from the capital, partners so far as may be necessary for the payment of d.) Those owing to partners in respect of partnership obligations to creditors and to partners. profits. A partner has a right to have debts owing to the 3.) The assets shall be applied in the order of partnership from his co-partners deducted from their their declaration in No. 1 of this article to respective shares. This right is called equitable lien or the satisfaction of the liabilities. quasi lien. It exists only when the affairs of the 4.) The partners shall contribute, as provided partnership are rounded up and the shares of the by Article 1797, the amount necessary to partners are computed after dissolution. satisfy the liabilities. 5.) An assignee for the benefit of creditors or Rules in settling accounts between partners after any person appointed by the court shall dissolution have the right to enforce the contributions Article 1839 sets forth a priority system for the specified in the preceding number. distribution of partnership property (see Art. 1810) and 6.) Any partner or his legal representative shall individual property when a partnership is dissolved. have the right to enforce the contributions The following rules are subject to variation by specified in No. 4, to the extent of the agreement of the partners. amount which he has paid in excess of his Assets of the partnership: share of the liability. 1.) Partnership property (including good will); and 7.) The individual property of a deceased 2.) Contributions of the partners necessary for the partner shall be liable for the contributions payment of all liabilities in accordance with specified in No. 4. Article 1797. 8.) When partnership property and the Order of application of the assets: individual properties of the partners are in 1.) Those owing to partnership creditors; possession of a court for distribution, 2.) Those owing to partners other than for capital partnership creditors shall have priority on and profits such as loans given by the partners partnership property and separate creditors or advances for business expenses; on individual property, saving the rights of 3.) Those owing for the return of the capital lien or secured creditors. contributed by the partners; and 9.) Where a partner has become insolvent or 4.) The share of the profits, if any, due to each his estate is insolvent, the claims against partner. his separate property shall rank in the Loans and advances made by partners to the following order: partnership are not capital. Nor are they undivided a.) Those owing to separate creditors; profit, unless otherwise agreed. Capital contributions are b.) Those owing to partnership returnable only on dissolution, but loans are payable at creditors; maturity and accumulated profits may be withdrawn at c.) Those owing to partners by way of any time by consent of a majority. contribution. Amounts paid into the partnership in excess of a partners agreed capital contributions constitute loans or KNOW THE RULES ON SETTLING ACCOUNTS. Its long advances which draw interest on which they were made. but it is simple Accumulated profits do not draw interest, as they are not regarded as loans and advances merely because Liquidation and distribution of assets of dissolved they are left with the firm. partnership Capital contributed by partners Capital The process of winding up, where the business of the represents a debt of the firm to the contributing dissolved partnership is not continued, consists in partners. If, on dissolution, partnership assets are reducing the property to cash and distributing the insufficient to repay capital investments, the deficit

is a proceeds. The property must be liquidated and capital loss which requires contribution like any other distributed. Partners severally have the implied authority loss. The return of the amount equivalent to the capital to sell partnership property and to collect obligations contribution of each partner shall be increased by his due to the partnership. These powers may be delegated share of undistributed profits or decreased by his share of net losses.
Helen C. Arevalo 39 Section 3D

Finals Reviewer PARTNERSHIP 1st Sem; 2003


A partner who furnishes no capital but contributes partnership property to one or more third merely his skill and services is not entitled to any part of persons who promise to pay the debts and the firm capital on dissolution in the absence of who continue the business of the dissolved agreement. He must look for his compensation to his partnership; share of the profits remaining after repayment of the 5.) When any partner wrongfully causes a capital to the contributors. dissolution and the remaining partners Right of a partner where assets insufficient If continue the business under the provisions the assets enumerated in No. 1 are insufficient, the of Article 1837, second paragraph, No. 2, deficit is a capital loss which requires contribution like either alone or with others, and without any other loss. Any partner or his legal representative, liquidation of partnership affairs; or any assignee for the benefit of the creditors or any 6.) When a partner is expelled and the person appointed by the court, shall have the right to remaining partners continue the business enforce the contributions. If any partner does not pay either alone or with others without his share, the others will pay but they can sue the non-liquidation of the partnership affairs. paying partner for indemnification. The liability of a third person becoming a Liability of deceased partners individual partner in the partnership continuing the business, property The individual property of a deceased under this article, to the creditors of the dissolved partner shall be liable for his share of the contributions partnership shall be satisfied out of the necessary to satisfy the liabilities of the partnership partnership property only, unless there is a incurred while he was a partner. stipulation to the contrary. Priority to payment of partnership When the business of a partnership after creditors/partners creditors Doctrine of dissolution is continued under any conditions set marshaling of assets: Partnership assets to partnership forth in this article the creditors of the dissolved creditors. Individual assets to individual creditors. partnership, as against the separate creditors of Anything left from either goes to the other. the retiring or deceased partner, have a prior right Distribution of property of insolvent partner If to any claim of the retired partner or the a partner is insolvent, his individual property shall be representative of the deceased partner against the distributed as follows: person or partnership continuing the business, on 1.) To those owing to separate creditors; account of the retired or deceased partners 2.) To those owing to partnership creditors; and interest in the dissolved partnership or on account 3.) To those owing to partners by way of of any consideration promised for such interest of contribution. for his right in partnership property. Nothing in this article shall be held to modify Art. 1840. In the following cases creditors of any right of creditors to set aside any assignment the dissolved partnership are also creditors of the on the ground of fraud. person of partnership continuing the business: The use by the person or partnership continuing 1.) When any new partner is admitted into an the business of the partnership name, or the name existing partnership, or when any partner of a deceased partner as part thereof, shall not of retires and assigns (or the representative of itself make the individual property of the deceased the deceased partner assigns) his rights in partner liable for any debts contracted by such partnership property to two or more of the person or partnership. partners, or to one or more of the partners and one or more third persons, if the NOT REALLY CODAL BUT COMMENTARY! business is continued without liquidation of the partnership affairs; Dissolution of a partnership by change in 2.) When all but one partner retire and assign membership (or the representative of a deceased partner Causes: assigns) their rights in partnership property 1.) New partner is admitted; to the remaining partner, who continues the 2.) Partner retires; business without liquidation of partnership 3.) Partner dies; affairs, either alone or with others; 4.) Partner withdraws; 3.) When any partner retires or dies and the 5.) Partner is expelled from partnership; business of the dissolved partnership is 6.) Other partners assign their rights to sole continued as set forth in Nos. 1 and 2 of this remaining partner; article, with the consent of the retired 7.) All the partners assign their rights in partnership partners or the representative of the property to 3rd persons. deceased partner, but without any Any change in membership dissolves a partnership

assignment of his right in partnership and creates a new one. property; Continuation of partnership without liquidation 4.) When all the partners or their A partnership dissolved by any of these happenings representatives assign their rights in need not undergo the procedure relating to dissolution
Helen C. Arevalo 40 Section 3D

Finals Reviewer PARTNERSHIP 1st Sem; 2003


and winding up of its business affairs. The remaining Exemption from liability of individual property of partners (and/or new partners) may elect to continue deceased partner the business of the old partnership w/o interruption by The last paragraph of Article 1840 primarily deals simply taking over the business enterprise owned by the with the exemption from liability to creditors of a preceding partner and continuing the use of the old dissolved partnership of the individual property of the name. The rights and obligations of the partners as deceased partner for debts contracted by the person or among themselves in case of such continuation are set partnership which continues eth business using the forth in Article 1837. It is, however, technically partnership name or name of the deceased partner as considered a new partnership. part thereof. What the law contemplates is a hold-over situation preparatory to formal reorganization. Rights of creditors of dissolved partnership which It treats more of a commercial partnership with a is continued good will to protect rather than a professional This article deals with the rights of creditors when the partnership with no saleable good will. partnership is dissolved by a change of membership and its business is continued. Art. 1841. When any partner retires or dies, and Equal rights of dissolved and new partnership the business is continued under any of the creditors The creditors of the old partnership are also conditions set forth in the preceding article, or in the creditors of the new partnership which continues the Article 1837, second paragraph, No. 2, without any business of the old one w/o liquidation of partnership settlement of accounts as between him or his affairs. estate and the person or partnership continuing Liability of persons continuing business Note the business, unless otherwise agreed, he or his that under par. 2, the liability of the new or incoming legal representative as against such person or partners shall be satisfied out of partnership property partnership may have the value of his interest at only unless there is a stipulation to the contrary. the date of dissolution ascertained, and shall Note that par. 1, No. 4, applies only when the 3rd receive as an ordinary creditor an amount equal to person continuing the business promises to pay the the value of his interest in the dissolved debts of the partnership. Otherwise, creditors of the partnership with interest, or at his option or at the dissolved partnership have no claim on the person or option of his legal representative, in lieu of partnership continuing the business or its property interest, the profits attributable to the use of his unless the assignment can be set aside as a fraud on right in the property of the dissolved partnership; creditors under par. 4. provided that the creditors of the dissolved Prior right of dissolved partnership creditors as partnership as against the separate creditors, or against purchaser When a retiring or deceased the representative of the retired or deceased partner has sold his interest in the partnership w/o a partner, shall have priority on any claim arising final settlement with creditors of the partnership, such under this article, as provided by Article 1840, creditors have an equitable lien on the consideration third paragraph. paid to the retiring or deceased partner by the purchaser thereof. This lien comes ahead of the claims of the Rights of retiring, or of estate of deceased, partner separate creditors of the retired or deceased partner. when business continued 1.) To have the value of the interest of the retiring Continuation of dissolved partnership business by partner or deceased partner in the partnership another company ascertained as of the date of dissolution (i.e. When corporation deemed a mere continuation date of retirement or death); and of prior partnership Where a corporation was 2.) To receive thereafter, as an ordinary creditor, an formed by, and consisted of, members of a partnership amount equal to the value of his share in the whose business and property was conveyed and dissolved partnership with interest, or, at his transferred to the corporation for the purpose of option, in lieu of interest, the profits attributable continuing business, in payment for which corporate to the use of his right. capital stock was issued, such corporation is presumed to have assumed partnership debts and is prima facie Art. 1842. The right to an account of his liable therefor. interest shall accrue to any partner, or his legal When obligations of company bought out representative as against the winding up partners

considered assumed by vendee When said or the surviving partners or the person or obligations are not of considerable amount or value partnership continuing the business, at the date of especially when incurred in the ordinary course, and dissolution, in the absence of any agreement to when the business of the latter is continued. (NOT when the contrary. the obligation is of extraordinary value, and the company was bought out to eliminate competition not EMPHASIZE! to continue business.) Accrual and prescription of a partners right to account of his interest
Helen C. Arevalo 41 Section 3D

Finals Reviewer PARTNERSHIP 1st Sem; 2003


The right to demand an accounting of the value of his such shall not be bound by the obligations of the interest accrues to any partner or his legal partnership. representative after dissolution in the absence of an agreement to the contrary. KNOW THE CHARACTERISTICS; DIFFERENCES BETWEEN Prescription beings to run only upon the dissolution GENERAL AND LIMITED PARTNERSHIP. of the partnership when the final accounting is done. Under Articles 1806, 1807, and 1809, the right to Concept of limited partnership demand an accounting exists as long as the partnership The form of business association composed of one or exists. more general partners and one or more special partners, the latter not being liable for the partnership debts. Person liable to render an account The liability of limited partners is limited to a fixed 1.) The winding up partner; amount their capital contributions or the amount they 2.) The surviving partner; or have invested in the partnership. 3.) The person or partnership continuing the business. Characteristics of limited partnership 1.) Formed by compliance with statutory Liquidation necessary for determination of requirements; partners share 2.) One or more general partners control the Share of the profits The profits of a business business and are personally liable to creditors; cannot be determined by taking into account the result 3.) One or more limited partners contribute to the of one particular transaction instead of all the capital and share in the profits but do not transactions had. Hence, the need for a general participate in the management of the business liquidation before a member of a partnership may claim and are not personally liable for partnership a specific sum as his share of the profits. obligations beyond the amount of their capital When there is lack of evidence to ascertain the contributions; profits for a given period of time, the average will be 4.) The limited partners may ask for the return of used of the time prior or subsequent thereto for which their capital contributions under the conditions there exists evidence. prescribed by law; and Share in the partnership A partners share 5.) The partnership debts are paid out of common cannot be returned w/o first dissolving and liquidating fund and the individual properties of the general the partnership, for the firms outside creditors have partners. preference over the assets of the enterprise and the firms property cannot be diminished to their prejudice. Business reason and purpose of statutes No specific amounts or properties may be adjudicated authorizing limited partnerships to the heir or legal representative of the deceased 1.) Secure capital from others for ones business and partner w/o the liquidation being first terminated. still retain control; 2.) Share in profits of a business without risk of When liquidation not required personal liability; As a general rule, when a partnership is dissolved, a 3.) Associate as partners with those having business partner or his legal representative is entitled to the skill payment of what may be due after a liquidation. But no liquidation is necessary when there is already a Differences between a general partner/ settlement or an agreement as to what he shall receive. partnership and a limited partner/partnership General partner Limited partner Personally liable for Liability extends only to his CHAPTER 4. LIMITED PARTNERSHIP partnership obligations. capital contribution. When the manner of mgt No share in the mgt of a Brief history has not been agreed upon, limited partnership. He Apparently even more ancient than ordinary all of the general partners becomes liable as a partnership. An outgrowth of Roman Lawschwar, have an equal right in the general partner if he takes schwar, schwar mgt of the business. part in control of business.

May contribute money, Must contribute cash or Sources of Civil Code provisions: Uniform Limited property, or industry. property. Not services. Partnership Act. Proper party to NOT a proper party in such proceedings by or against proceedings. Article 1843. A limited partnership is one partnership. formed by two or more persons under the His interest cannot be His interest is freely provisions of the following article, having as assigned as to make the assignable, with the members one or more general partners and one or assignee a new partner assignee acquiring all the more limited partners. The limited partners as w/o the consent of the rights of a limited partner other partners. subject to certain
Helen C. Arevalo 42 Section 3D

Finals Reviewer PARTNERSHIP 1st Sem; 2003


qualifications. contributor in his place, and the His name may appear in Generally, his name must terms and conditions of the the firm name. not. substitution; Prohibited from engaging No such prohibition. k.) The right, if given, of partners to in a business which is of admit additional limited partners; the kind of business in l.) The right, if given, of one or more of which the partnership is the limited partners to priority over engaged (if capitalist) or other limited partners, as to any business for himself (if contributions or as to compensation industrial). by way of income, and the nature of His retirement, death, His retirement, death, such priority; insanity, or insolvency insanity, insolvency does m.) The right, if given, of the remaining dissolves the partnership. not. general partner or partners to continue the business on the death, The above also indicate the differences between a retirement, civil interdiction, insanity general partnership and a limited partnership. The other or insolvency of a general partner; differences are: and General partnership Limited Partnership n.) The right, if given, of a limited May be constituted in any Created by the members partner to demand and receive form by contract or after compliance with the property other than cash in return conduct of the parties. requirements set forth by for his contribution. law. 2.) File for record the certificate in the Office of Composed only of general Composed both of general the Securities and Exchange Commission. partners. and limited partners. A limited partnership is formed if there has Any firm name desired as Must operate under a firm been substantial compliance in good faith with the long as not same, name followed by the word foregoing requirements. confusingly similar. Limited. NOT CODAL BUT COMMENTARY. Art. 1844. Two or more persons desiring to form a limited partnership shall: Limited partnership not created by mere voluntary 1.) Sign and swear to a certificate, which shall agreement state A limited partnership is formed if there has been a.) The name of the partnership, adding substantial compliance in good faith with the thereto the word Limited; requirements set forth in Article 1844. b.) The character of the business; The creation of a limited partnership is a formal c.) The location of the principal place of proceeding and is not a mere voluntary agreement, as in business; the case of a general partnership. Accordingly, the d.) The name and place of residence of requirements of the statute must be followed; each member, general and limited otherwise, the liability of the limited partners becomes partners being respectively the same as that of general partners. designated; e.) The term for which the partnership is Requirements for formation of a limited to exist; partnership f.) The amount of cash and a description 1.) The certificate or articles of the limited of and the agreed value of the other partnership which states the matters property contributed by each limited enumerated in the article, must be signed and partner; sworn to; and g.) The additional contributions, if any, 2.) Such certificate must be filed for record in the to be made by each limited partner Office of the SEC. and the times at which or events on the happening of which they shall be Execution of the prescribed certificate made; A prime requisite to the formation of a limited h.) The time, if agreed upon, when the partnership, under Article 1844, is the execution of the contribution of each limited partner prescribed certificate. This document, as a rule, must is to be returned; contain the matters enumerated in said article. Thus, a i.) The share of the profits or the other limited partnership cannot be constituted orally. compensation by way of income This is to protect those who deal with the firm.

which each limited partner shall The statements required in the certificate must be receive by reason of his contribution; true at the time the certificate and other required papers j.) The right, if given, of a limited are filed with the SEC. partner to substitute an assignee as
Helen C. Arevalo 43 Section 3D

Finals Reviewer PARTNERSHIP 1st Sem; 2003


A person who files a false certificate thereby renders Medium A limited partner is not allowed to himself liable as a general partner. contribute services. He can contribute only money or property; otherwise, he shall be considered as industrial Substantial compliance in good faith sufficient and general partner, in which case, he shall not be Rules applicable where there is no substantial exempted from personal liability. compliance The firm becomes a general partnership A partner may be a general partner and a limited only as to its relation to 3rd persons. It is, in form, still a partner in the same partnership at the same time, limited partnership subject to all the rules applicable to a provided that this fact shall be stated in the certificate, limited partnership. Thus, a limited partner treated as a but a limited partner may not be an industrial partner general partner as far as 3rd persons are concerned is without being a general partner. entitled to reimbursement from the general partner for Time The contribution of each limited partner must whatever obligations he might have paid to partnership be paid before the formation of the limited partnership, creditors beyond his capital contribution. although with respect to the additional contributions Rule where partnership creditor guilty of they may be paid after the limited partnership has been estoppel If attaching creditors recognize and deal formed. with a firm as a limited partnership, they will be estopped from insisting that there is no such Art. 1846. The surname of a limited partner partnership, or that the terms of the partnership were shall not appear in the partnership name unless: not sufficiently stated in the notice of its formation. 1.) It is also the surname of a general partner; or Presumption of general partnership 2.) Prior to the time when the limited partner A partnership transacting business is, prima facie, a became such, the business had been carried general partnership and those who seek to avail on under a name in which his surname had themselves of the protection of laws permitting the appeared. creation of limited partnerships must show due A limited partner whose surname appears in a compliance with such laws. partnership name contrary to the provisions of the first paragraph is liable as a general partner to Construction of provisions on limited partnerships partnership creditors who extend credit to the General rule: courts should adopt and enforce a partnership without actual knowledge that he is construction which, on the one hand, will not defeat one not a general partner. of the objects of the law and, upon the other hand, will not under cover of a substantial compliance with the Effect where surname of limited partner appears in requirements with the requirements of the statute, partnership name fritter away the protection which the law has thrown Limited partner becomes liable to partnership around persons dealing with such partnerships. creditors w/o the rights of a general partner. Of course, Courts must consider substance rather than form in such limited partner shall not be liable as a general construing the law. However, it should be construed to partner w/ respect to 3rd persons w/ actual knowledge insure substantial compliance with all the statutory that he is only a limited partner. provisions which are designed for the protection of persons dealing with the partnership. Art. 1847. If the certificate contains false statements, one who suffers loss by reliance on Who may become limited partners such statement may hold liable any party to the Under a statute which provides that the membership certificate who knew the statement to be false: of a limited partner consists of specified persons, a 1.) At the time he signed the certificate; or partnership cannot become a limited partner. An 2.) Subsequently, but within a sufficient time existing general partnership may be changed into a before the statement was relied upon to limited one, and a partner in the former general enable him to cancel or amend the partnership may become a limited partner in the limited certificate, or to file a petition for its partnership thus formed. cancellation or amendment as provided in

Article 1865. Art. 1845. The contributions of a limited partner may be cash or other property, but not services. Liability for false statement in certificate Under this provision, any partner to the certificate EMPHASIZE! containing a false statement is liable provided the following requisites are present: Limited partners contribution 1.) He knew the statement to be false at the time he signed the certificate, or subsequently, but having sufficient time to cancel or amend it or file a petition for its cancellation or amendment, he failed to do so;
Helen C. Arevalo 44 Section 3D

Finals Reviewer PARTNERSHIP 1st Sem; 2003


2.) The person seeking to enforce liability has relied Admission of additional limited partners upon the false statement in transacting business After a limited partnership has been formed, with the partnership; and additional limited partners may be admitted, provided 3.) The person suffered loss as a result of reliance there is proper amendment to the certificate which must upon such false statement. be signed and sworn to by all the partners, including the new limited partners, and filed with the SEC. Art. 1848. A limited partner shall not become liable as a general partner unless, in addition to Art. 1850. A general partner shall have the the exercise of his rights and powers as a limited rights and powers and be subject to all the partner, he takes part in the control of the restrictions and liabilities of a partnership without business. limited partners. However, without the written consent or ratification of the specific act by all the EMPHASIZE! limited partners, a general partner or all of the general partners have no authority to: Liability of limited partner for participating in 1.) Do any act in contravention of the management of partnership certificate; A limited partner is liable as a general partner for the 2.) Do any act which would make it impossible firms obligations if he takes part or interfere in the to carry on the ordinary business of the management of the firms business. Bare grant of partnership; apparent control does not make him liable where he has 3.) Confess a judgment against the partnership; not actually participated in the control of the 4.) Possess partnership property, or assign partnership. their rights in specific partnership property, Whether the limited partner has participated in the for other than a partnership purpose; management is to be determined by whether he has 5.) Admit a person as a general partner; exercised a controlling power in the firms transactions. 6.) Admit a person as a limited partner, unless the right so to do is given in the certificate; Active management of partnership business 7.) Continue the business with partnership contemplated property on the death, retirement, insanity, Such control contemplates active participation in the civil interdiction or insolvency of a general management of the partnership business and does not partner, unless the right so to do is given in comprehend the mere giving of advice to general the certificate. partners. The limited partner takes part in the management of EMPHASIZE! the business and is liable generally for the firms obligations where: Rights, powers, and liabilities of a general partner 1.) The business of the partnership is in fact carried The essential feature of a limited partnership is the on by a board of directors chosen by the limited union of 2 classes of members limited and general partners; partners. The law expressly requires there to be at least 2.) By the terms of the contract between the one general partner with unlimited liability. parties, an appointee of the limited partner Right of control/unlimited personal liability A becomes the directing manager of the firm; general partner in a limited partnership is vested with 3.) The limited partner purchases the entire the entire control of the firms business and has all the property of the partnership, taking title in rights and powers and is subject to all the liabilities and himself and then carries on the business in his restrictions of a partner in a partnership without limited own name and for his own exclusive benefit; or partners, i.e., in a general partnership. It is in 4.) He makes or is a party to a contract with consideration of his unlimited personal liability for the creditors of an insolvent firm with respect to the obligation of the partnership that he is granted the disposal of the firms assets in payment of the general authority to manage the firms business. firms debts. Acts of administration/acts of strict dominion The interference contemplated is with respect to an As a rule, he may bind the partnership by any act of existing limited partnership. A limited partner is not administration, but he has no power to do the specific subject to general liability for taking part in the acts enumerated in Article 1850 (even if agreed to by all

management of the firm because he settles its affairs the general partners) without the written consent or at after dissolution. least ratification of all the limited partners. The said acts are acts of strict dominion or ownership and are, Art. 1849. After the formation of a limited therefore, beyond the scope of the authority of a general partnership, additional limited partners may be partner. admitted upon filing an amendment to the original The general partner who violates the requirement certificate in accordance with the requirements of imposed by Article 1850 is liable for damages to the Article 1865. limited partners.

Helen C. Arevalo 45 Section 3D

Finals Reviewer PARTNERSHIP 1st Sem; 2003


Other limitations The general partners, of course, Art. 1852. Without prejudice to the provisions have no power to bind the limited partners beyond the of article 1848, a person who has contributed to latters investment. Neither do they have the power to the capital of the business conducted by a person act for the firm beyond the purpose and scope of the or partnership erroneously believing that he has partnership, and they have no authority to change the become a limited partner in a limited partnership, nature of the business w/o the consent of the limited is not, by reason of his exercise of the rights of a partners. limited partner, a general partner with the person or in the partnership carrying on the business, or Art. 1851. A limited partner shall have the same bound by the obligations of such person or rights as a general partner to: partnership; provided that on ascertaining the 1.) Have the partnership books kept at the mistake he promptly renounces his interest in the principal place of business of the profits of the business or other compensation by partnership, and at a reasonable hour to way of income. inspect and copy any of them; 2.) Have on demand true and full information of Status of partner where there is failure to create all things affecting the partnership, and a limited partnership formal account of partnership affairs A limited partnership is formed when there is whenever circumstances render it just and substantial compliance in good faith with the reasonable; and requirements of the law. If not complied with, the 3.) Have dissolution and winding up by decree limited partner will have the liability of a general partner of court. as to 3rd persons. Article 1852 provides for an A limited partner shall have the right to receive exemption. a share of the profits or other compensation by way of income, and to the return of his Status of person erroneously believing himself to contribution as provided in articles 1856 and be a limited partner 1857. Conditions for exemption from liability as general partner: EMPHASIZE! 1.) On ascertaining the mistake, he promptly renounces his interest in the profits of the Rights, in general, of a limited partner business or other compensation by way of The limited partner is viewed as a partner only to a income; certain extent. His powers, actual or implied, are much 2.) His surname does not appear in the partnership more limited than those of a general partner. As name; and between the members of the firm, the limited partner, in 3.) He does not participate in the management of order to protect his interest in the firm, has the same the business. right to compel the partners to account as a general Necessity of renouncing his interest The partner has. person must promptly renounce his interest before the Wrongdoing or improper acts on the part of general partnership has become liable to 3rd persons who cannot partners may not give a limited partner greater rights be blamed for considering him a general partner. Where than the law and what his contract grants him. no partnership creditors are prejudiced, it would seem that renunciation of his interest is not necessary. Specific rights of a limited partner Obligation to pay back profits and 1.) To require that the partnership books be kept at compensation already received There are two the principal place of business of the opinions as to whether the limited partner should also partnership; return profits and compensation already received. Some 2.) To inspect and copy at a reasonable hour say yes, some say no. Both views have merit. partnership books or any of them; 3.) To demand true and full information of all things Status of heirs of a deceased general partner affecting the partnership; admitted as partners 4.) To demand a formal account of partnership An heir of a deceased general partner admitted as a affairs whenever circumstances render it just partner ordinarily becomes a limited partner. He may, and reasonable; however, choose to become a general partner if he so 5.) To ask for dissolution and winding up by decree wishes. The right of an heir of a deceased partner

to be of court; admitted as a partner, when provided in the articles of 6.) To receive a share of the profits or other partnership, can be waived by the heir. He cannot be compensation by way of income; and compelled to become a partner. It is his choice. 7.) To receive the return of his contribution provided the partnership assets are in excess of all its Art. 1853. A person may be a general partner liabilities. and a limited partner in the same partnership at the same time, provided that this fact shall be stated in the certificate provided for in Art. 1844.
Helen C. Arevalo 46 Section 3D

Finals Reviewer PARTNERSHIP 1st Sem; 2003


A person who is a general, and also at the same 2.) Receiving any payment, conveyance, or release time a limited partner, shall have all the rights and from liability if it will prejudice the right of 3rd powers and be subject to all the restrictions of a persons. general partner; except that, in respect to his Any violation of the prohibition will give rise to the contribution, he shall have the rights against the presumption that it has been made to defraud other members which he would have had if he partnership creditors. were not also a general partner. Preferential rights of third persons In transacting business with the partnership as a nonOne person, both a general partner and a limited member, the limited partner is considered as a nonpartner partner creditor. However, 3rd persons always enjoy A person may be a general and a limited partner at preferential rights insofar as partnership assets are the same time in the same partnership provided that concerned in view of the natural tendency of the this fact is stated in the certificate signed, sworn to, and partners to give preference to each other. recorded in the Office of the Securities and Exchange Commission. Art. 1855. Where there are several limited Generally. His rights and powers are those of a partners the members may agree that one or more general partner. Hence, he is liable with his separate of the limited partners shall have priority over property to 3rd persons. However, with respect to his other limited partners as to the return of their contribution as a limited partner, he would have the contributions, as to their compensation by way of right of a limited partner insofar as the other partners income, or as to any other matter. If such an are concerned. This means that while he is not relieved agreement is made, it shall be stated in the from personal liability to 3rd persons for partnership certificate, and in the absence if such a statement, debts, he is entitled to recover from the general all the limited partners shall stand upon equal partners the amount he has paid to such 3rd persons; footing. and in settling accounts after dissolution, he shall have priority over general partners in the return of their Preferred limited partners respective contributions. By an agreement of all the members (general and limited) stated in the certificate, priority may be given to Art. 1854. A limited partner also may loan some limited partners over other limited partners as to money to and transact other business with the the: partnership, and, unless he is also a general 1.) Return of their contributions; partner, receive on account of resulting claims 2.) Their compensation by way of income; or against the partnership, with general creditors a 3.) Any other matter. pro rata share of the assets. No limited partner In the absence of such statement in the certificate, shall in respect to any such claim: even if there is an agreement, all the limited partners 1.) Receive or hold as collateral security any shall stand on equal footing in respect to these matters. partnership property; or 2.) Receive from a general partner or the Art. 1856. A limited partner may receive from partnership any payment, conveyance or the partnership the share of the profits or the release from liability, if at the time the compensation by way of income stipulated for in assets of the partnership are not the certificate; provided, that after such payment sufficient to discharge partnership is made, whether from the property of the liabilities to persons as general or limited partnership or that of a general partner, the partners. partnership assets are in excess of all liabilities to The receiving of collateral security, or a limited partners on account of their contributions payment, conveyance, or release in violation of the and to general partners. foregoing provisions is a fraud on the creditors of the partnership. Compensation of limited partner The right of the limited partner to receive his share of Loan and other business transactions with limited the profits or compensation by way of income stipulated partnership for in the certificate is subject to the condition that Allowable transactions: partnership assets will still be in excess of partnership 1.) Granting loans to the partnership; liabilities after such payment. In other words, 3rd party 2.) Transacting other business with it; creditors have priority over the limited partners rights. 3.) Receiving a pro rata share of the partnership In determining the liabilities of the partnership, the assets with general creditors if he is not also a liabilities to the limited partners for their contribution

general partner. and to general partners, whether for contributions or Prohibited transactions: not, are not considered. 1.) Receiving or holding as collateral security any partnership property; or
Helen C. Arevalo 47 Section 3D

Finals Reviewer PARTNERSHIP 1st Sem; 2003


Art. 1857. A limited partner shall not receive 3.) After the expiration of the 6 months notice in from a general partner or out of partnership writing given by him to the other partners if no property any part of his contributions until: time is fixed in the certificate for the return of the 1.) All liabilities of the partnership, except contribution or for the dissolution of the liabilities to general partners and to partnership. limited partners on account of their contributions, have been paid or there Right of limited partner to cash in return for remains property of the partnership contribution sufficient to pay them; Under the 3rd paragraph, even if a limited partner has 2.) The consent of all members is had, unless contributed property, he has only the right to demand the return of the contribution may be and receive cash for his contribution. Exceptions: rightfully demanded under the provisions 1.) When there is stipulation to the contrary in the of the second paragraph; and certificate; or 3.) The certificate is cancelled or so amended 2.) Where all the partners (general and limited) as to set forth the withdrawal or consent to the return other than in the form of reduction. cash. Subject to the provisions of the first paragraph, a limited partner may rightfully demand the return When limited partner may have partnership of his contribution: dissolved 1.) On the dissolution of a partnership; or The 4th paragraph provides for additional grounds for 2.) When the date specified in the certificate for the dissolution of the partnership upon petition of a its return has arrived; or limited partner: 3.) After he has given six months notice in 1.) When his demand for the return of his writing to all other members, if no time is contribution is denied although he has a right to specified in the certificate, either for the such return; or return of the contribution or for the 2.) When his contribution is not paid although he is dissolution of the partnership. entitled to its return because the other liabilities In the absence of any statement in the of the partnership have not been paid or the certificate to the contrary or the consent of all partnership property insufficient for their members, a limited partner, irrespective of the payment. nature of his contribution, has only the right to The limited partner must first ask the other partners demand and receive cash in return for his to have the partnership dissolved; if they refuse, then contributions. he can seek the dissolution of the partnership by judicial A limited partner may have the partnership decree. dissolved and its affairs wound up when; 1.) He rightfully but unsuccessfully demands Art. 1858. A limited partner is liable to the his return of his contribution; or partnership: 2.) The other liabilities of the partnership have 1.) For the difference between his contribution not been paid, or the partnership property is as having been made, and insufficient for their payment as required by 2.) For any unpaid contribution which he the first paragraph, No. 1, and the limited agreed in the certificate to make in the partner would otherwise be entitled to the future at the time and on the conditions return of his contribution. stated in the certificate. A limited partner holds as trustee for the Requisites for return of contribution of limited partnership: partner 1.) Specific property stated in the certificate as 1.) All liabilities of the partnership have been paid or contributed bv him, but which was not if they have not yet been paid, the assets of the contributed or which has been wrongfully partnership are sufficient to pay such liabilities; returned, and 2.) The consent of all members (general and 2.) Money or other property wrongfully paid or limited) has been obtained except when the conveyed to him on account of his retuirn may be rightfull demanded; and contribution. 3.) The certificate is cancelled or so amended as to The liabilities of a limited partner as set forth in set forth the withdrawal or reduction of the this article can be waived or compromised only by contribution. the consent of all members; but a waiver or compromise shall not affect the right of a creditor When return a matter of right of a partnership who extended credit or whose 1.) On the dissolution of the partnership; or claim arose after the filing and before a

2.) Upon arrival of the date specified in the cancellation or amendment of the certificate, to certificate for the return; or enforce such liabilities.
Helen C. Arevalo 48 Section 3D

Finals Reviewer PARTNERSHIP 1st Sem; 2003


When a contributor has rightfully received the The limited partner is liable to the partnership for the return in whole or in part of the capital of his return of contribution lawfully received by him to pay contribution, he is nevertheless liable to the creditors who extended credit or whose claim arose partnership for any sum, not in excess of such before such return. His liability, of course, cannot return with interest, necessary to discharge its exceed the sum received by him with interest. liabilities to all creditor who extended credit or whose claims arose before such return. Art. 1859. A limited partners interest is assignable. DEINS NA TO! A substituted limited partner is a person admitted to all the rights of a limited partner who Liabilities of a limited partner has died or has assigned his interest in a To the partnership Liability of limited partners is partnership. to partnership, not the creditors of the partnership. An assignee, who does not become a To partnership creditors and other partners A substituted limited partner, has no right to require limited partner is liable for partnership obligations when: any information or account of the partnership 1.) Contributes services; transactions or to inspect the partnership books; 2.) Allows his surname to appear in name of firm; he is only entitled to receive the share of the 3.) Fails to have false statement in certificate profits or other compensation by way of income, corrected when he knew it to be false; or the return of his contribution, to which his 4.) Takes part in control of business; assignor would otherwise be entitled. 5.) Receives partnership property as collateral An assignee shall have the right to become a security, payment, conveyance, or release in substituted limited partner if all the members fraud of partnership creditors; consent thereto or if the assignor, being thereunto 6.) Failure to substantially comply with legal empowered by the certificate, gives the assignee requirements of formation of limited partnership. that right. To separate creditors Creditor of limited partner An assignee becomes a substituted limited may also apply for a charging order subjecting the partner when the certificate is appropriately interest in the partnership of the debtor partner for the amended in accordance with article 1865. payment of his obligation. The substituted limited partner has all the rights and powers, and is subject to all the Liability for unpaid contribution restrictions and liabilities of his assignor, except Limited partner liable not only for the difference those liabilities of which he was ignorant at the between the amount of his actual contributions and that time he became a limited partner and which could stated in the certificate as having been made but also not be ascertained for the certificate. for any unpaid contribution he agreed to make at a The substitution of the assignee as a limited future time. partner does not release the assignor from liability to the partnership under Articles 1847 and 1858. Liability as trustee Limited partner considered as trustee for the DEINS NA TO! partnership for: 1.) Specific property stated in the certificate as Effect of change in the relation of limited partners contributed by him but which he had not Does not necessarily dissolve the partnership. No contributed; limited partner, however, can withdraw his contribution 2.) Specific property of the partnership which had until all liabilities to creditors are paid. been wrongfully returned to him; 3.) Money wrongfully paid or conveyed to him on Rights of assignee of limited partner account of his contribution; and Assignee is only entitled to receive the share of the 4.) Other property wrongfully paid or conveyed to profits or other compensation by way of income or the him on account of his contribution. return of the contribution to which the assignor would otherwise be entitled. He has no right to require any Requisites for waiver or compromise of liabilities information or account of the partnership transactions or 1.) Waiver or compromise is made with the consent to inspect partnership books. of all the partners; and The assignee acquires all the rights of the limited 2.) The waiver or compromise does not prejudice partner only when he becomes a substituted limited partnership creditors who extended credit or partner. whose claims arose before the cancellation or

amendment of the certificate. When assignee may become substituted limited partner Liability for return of contribution lawfully Requisites: received 1.) All the members must consent to the assignee becoming a substituted limited partner or the
Helen C. Arevalo 49 Section 3D

Finals Reviewer PARTNERSHIP 1st Sem; 2003


limited partner, being empowered by the The remedies conferred by the first paragraph certificate, must give the assignee the right to shall not be deemed exclusive of others which may become a limited partner; exist. 2.) The certificate must be amended; Nothing in this Chapter shall be held to deprive 3.) The certificate as amended must be registered in a limited partner of his statutory exemption. the SEC. Rights of creditors of limited partner Liability of substituted partner and assignor Apply to court for charging order on limited partners Substituted limited partner is liable for all the interest in the partnership. liabilities of his assignor except only those of which he was ignorant at the time he became a limited partner Art. 1863. In settling accounts after dissolution and which could not be ascertained from the certificate. the liabilities of the partnership shall be entitled to payment in the following order: Art. 1860. The retirement, death, insolvency, 1.) Those to creditors, in the order of priority insanity or civil interdiction of a general partner as provided by law, except those to limited dissolves the partnership, unless the business is partners on account of their contributions, continued by the remaining general partners: and to general partners; 1.) Under a right so to do stated in the 2.) Those to limited partners in respect to their certificate, or share of the profits and other compensation 2.) With the consent of all members. by way of income on their contributions; 3.) Those to limited partners in respect to the Effect of retirement, death, etc. of a general capital of their contributions; partner 4.) Those to general partners other than for Dissolution of partnership. If limited partner, does capital and profits; not dissolve partnership unless he is the only limited 5.) Those to general partners in respect to partner. profits; If the business is continued by the remaining 6.) Those to general partners in respect to partners under the rights given in the certificate or with capital. the consent of all members, no dissolution but certificate Subject to any statement in the certificate or to must be amended for limited partners to still avail of subsequent agreement, limited partners share in limited liability. the partnership assets in respect to their claims for capital, and in respect to their claims for Art. 1861. On the death of a limited partner his profits or for compensation by way of income on executor or administrator shall have all the rights their contribution respectively, in proportion to the of a limited partner for the purpose of settling his respective amounts of such claims. estate, and such power as the deceased had to constitute his assignee a substituted limited EMPHASIZE! partner. The estate of a deceased limited partner shall Dissolution of a limited partnership liable for all his liabilities as a limited partner. Causes: 1.) Misconduct of a general partner; Right of executor on death of a limited partner 2.) Fraud practiced on the limited partner by the 1.) All the rights for purposes of settling the affairs general partner; of the limited partner; and 3.) Retirement, death, etc. of a general partner; 2.) The right to constitute the deceaseds assignee 4.) When all the limited partners ceased to be such; as substituted limited partner (if deceased was 5.) Expiration of the term for which partnership was empowered to so assign under certificate). to exist; or 6.) Mutual consent of the partners before the Art. 1862. On due application to a court of expiration of the firms original term. competent jurisdiction by any creditor of a limited Suit for dissolution A limited partner may bring a partner, the court may charge the interest of the suit for the dissolution of the firm, an accounting, and indebted limited partner with payment of the the appointment of a receiver when the misconduct of a unsatisfied amount of such claim, and may appoint general partner or the insolvency of the firm warrants it. a receiver, and make all other orders, directions, Similarly, creditors of a limited partnership are entitled and inquiries which the circumstances of the case to such relief where the firm is insolvent. may require. A limited partner may have the partnership dissolved The interest may be redeemed with the and its affairs wound up when he rightfully but separate property of any general partner, but may unsuccessfully demands the return of his contribution, or

not be redeemed with partnership property. the other liabilities of the partnership, except liabilities to general partners and to limited partners on account of their contributions, have not been paid, or the
Helen C. Arevalo 50 Section 3D

Finals Reviewer PARTNERSHIP 1st Sem; 2003


partnership property is insufficient for their payment, becomes controlling as between the partners and the limited partner would otherwise be entitled to themselves. In the absence of any contrary agreement, the return of his contribution. all the limited partners stand upon equal footing. Notice of dissolution When the firm is dissolved The claims of limited partners for profits and other by the expiration of the term fixed in the certificate, compensation by way of income and return of capital notice of the dissolution need not be given since the contributions rate ahead with respect to all claims of papers filed and recorded in the SEC are notice to all the general partners. For claims arising from individual loans world of the term of the partnership. Where, however, to, or other business transactions with, the partnership, the dissolution is by the express will of the partners, the other than for capital contributions, the limited partner certificate shall be canceled, and a dissolution of the is placed in the same category as a non-member partnership is not effected until there has been creditor. If return is made to a limited partner of his compliance with the requirement in this respect. contribution before creditors are paid, he is under an Winding up When a limited partnership has been obligation to reimburse such payments, with interest, so duly dissolved, the general partners have the right and far as necessary to satisfy claims of creditors. power to wind up its affairs. It is not the duty of the In the event of insolvency of the partnership, its limited partner or of the representatives of a deceased creditors take preference over both general and limited limited partner to care for or collect the assets of the partners. firm. Art. 1864. The certificate shall be cancelled Priority in the distribution of partnership assets when the partnership is dissolved or all limited The partnership liabilities shall be settled in the partners cease to be such. following order: A certificate shall be amended when: 1.) Those due to creditors, including limited 1.) There is a change in the name of the partners, except those on account of their partnership or in the amount or character contributions, in the order of priority as provided of the contribution of any limited partner; by law; 2.) A person is substituted as a limited partner; 2.) Those due to limited partners in respect to their 3.) An additional limited partner is admitted; share of the profits and other compensation by 4.) A person is admitted as a general partner; way of income on their contributions; 5.) A general partner retires, dies, becomes 3.) Those due to limited partners for the return of insolvent or insane, or is sentenced to civil the capital contributed; interdiction and the business is continued 4.) Those due to general partners other than that under Article 1860; for capital and profits; 6.) There is a change in the character of the 5.) Those due to general partners in respect to business of the partnership; profits; and 7.) There is a false or erroneous statement in 6.) Those due to general partners for the return of the certificate; the capital contributed. 8.) There is a change in the time as stated in Partnership creditors are entitled to first distribution, the certificate for the dissolution of the followed by limited partners who take priority over partnership or for the return of a general partners. contribution; Note that in a general partnership, the claims of the 9.) A time is fixed for the dissolution of the general partners in respect of capital enjoy preference partnership, or the return of a contribution, over those in respect of profits. no time having been specified in the certificate; or Shares of limited partners in partnership assets 10.) The members desire to make a change in In the absence of any statement in the certificate as any other statement in the certificate in to the share of the profits which each partner shall order that it shall accurately represent the receive by reason of his contribution and subject to any agreement among them. subsequent agreement, limited partners share in the

partnership assets in respect to their claims for capital EMPHASIZE! and profits in proportion to the respective amounts of such claims. When certificate shall be cancelled or amended This proportional sharing by the limited partners The certificate shall be cancelled, not merely takes place where the partnership assets are insufficient amended: to pay such claims. 1.) When the partnership is dissolved other than by reason of the expiration of the term of the Priority of claims of limited partners partnership; or The members of a limited partnership, as among 2.) When all the limited partners cease to be such. themselves, may include in the partnership articles an A limited partnership cannot exist as such if agreement for priority of distribution on the winding up there are no more limited partners. of partnership affairs. Such agreement ordinarily
Helen C. Arevalo 51 Section 3D

Finals Reviewer PARTNERSHIP 1st Sem; 2003


In other cases, only an amendment of the certificate Requirements to cancel: same. If cancellation is is required. ordered by the court, certified copy of such order shall be filed with the SEC. Art. 1865. The writing to amend a certificate Approval by Commission is not required for either shall: case. 1.) Conform to the requirements of Article 1844 as far as necessary to set forth clearly Art. 1866. A contributor, unless he is a general the change in the certificate which it is partner, is not a proper party to proceedings by or desired to make; and against a partnership, except where the object is 2.) Be signed and sworn to by all members, to enforce a limited partners right against or and an amendment substituting a limited liability to the partnership. partner or adding a limited or general partner shall be signed also by the member DEINS NA DIN TO! to be substituted or added, and when a limited partner is to be substituted, the Limited partner, a mere contributor amendment shall also be signed by the A limited partner is a mere contributor. He is assigning limited partner. practically a stranger in the limited partnership whose The writing to cancel a certificate shall be liability is limited to his interest in the firm, without any signed by all members. right and power to participate in the management and A person desiring the cancellation or control of the business. Relationship between limited amendment of a certificate, if any person partner and partnership is not one of trust and designated in the first and second paragraphs as a confidence. person who must execute the writing refuses to do so, may petition the court to order a cancellation Parties to action by or against partnership or amendment thereof. Since limited partners are not principals in If the court finds that the petitioner has a right partnership transactions, their liability, as a general rule, to have the writing executed by a person who is to the partnership, not the creditors of the refuses to do so, it shall order the Office of the partnership. For the same reason, they have no right of Securities and Exchange Commission where the action against 3rd persons against whom the partnership certificate is recorded, to record the cancellation has any enforceable claim. or amendment of the certificate; and when the certificate is to be amended, the court shall also When limited partner a proper party cause to be filed for record in said office a certified 1.) Where the object is to enforce limited partners copy of its decree setting forth the amendment. individual rights against the partnership, and to A certificate is amended or cancelled when recover damages for violation of such right. there is filed for record in the Office of the 2.) When its a proceeding to enforce his liability to Securities and Exchange Commission, where the the partnership; certificate is recorded: 3.) Creditors may go against him if he had 1.) A writing in accordance with the withdrawn sums from the capital of the firm with provisions of the first or second outstanding debts on a voluntary dissolution. paragraph; or 2.) A certified copy of the order of court in Nature of limited partners interest in firm accordance with the provisions of the 1.) Limited partners contributions are not a loan, fourth paragraph; and he is not a creditor of the firm because of 3.) After the certificate is duly amended in such contribution; accordance with this article, the amended 2.) Limited partners contribution is not a mere certificate shall thereafter be for all investment; purposes the certificate provided for in 3.) Limited partner is, in a sense, an owner, which in this Chapter. interest in the capital of the firm and its business as such, but he has no property right in the firms DEINS NA TO! assets; 4.) Limited partners interest is in personal property; Requirements for amendment and cancellation of 5.) The nature of the limited partners interest in the certificate firm amounts to a share in the partnership assets Requirements to amend: after its liabilities have been deducted and a 1.) Amendment must be in writing; balance struck. This interest in a chose in action, 2.) It must be signed and sworn to by all the and hence, intangible personal property.

members; and 3.) The certificate, as amended, must be filed for Art. 1867. A limited partnership formed under record in the SEC. the law prior to the effectivity of this Code, may become a limited partnership under this Chapter
Helen C. Arevalo 52 Section 3D

Finals Reviewer PARTNERSHIP 1st Sem; 2003


by complying with the provisions of Article 1844, provided the certificate sets forth: 1.) The amount of the original contribution of each limited partner, and the time when the contribution was made; and 2.) That the property of the partnership exceeds the amount sufficient to discharge its liabilities to persons not claiming as general or limited partners by an amount greater than the sum of the contributions of its limited partners. A limited partnership formed under the law prior to the effectivity of this Code, until or unless it becomes a limited partnership under this Chapter, shall continue to be governed by the provisions of the old law. PATI TO DEINS NA! Provisions for existing limited partnerships A limited partnership formed under the former law may become a limited partnership by complying with the provisions of Article 1844, provided the certificate sets forth the information required by Article 1867. Until or unless it becomes a limited partnership under this chapter, it shall continue to be governed by the provisions of the old law. Good Luck!

Helen C. Arevalo 53 Section 3D

You might also like